Why is there something rather than nothing? From quantum theory to dialectics?

485 views
Skip to first unread message

Peter Sas

unread,
Oct 22, 2014, 4:33:50 AM10/22/14
to everyth...@googlegroups.com
Hi guys,

Here is a blog piece I wrote about nothing as the ultimate source of being:

http://critique-of-pure-interest.blogspot.nl/2014/09/why-is-there-something-rather-than.html

Richard Ruquist

unread,
Oct 22, 2014, 9:46:16 AM10/22/14
to everyth...@googlegroups.com
Peter,

Could you elaborate on how Dark Energy fits into your thesis?
Richard

--
You received this message because you are subscribed to the Google Groups "Everything List" group.
To unsubscribe from this group and stop receiving emails from it, send an email to everything-li...@googlegroups.com.
To post to this group, send email to everyth...@googlegroups.com.
Visit this group at http://groups.google.com/group/everything-list.
For more options, visit https://groups.google.com/d/optout.

meekerdb

unread,
Oct 22, 2014, 11:33:17 AM10/22/14
to everyth...@googlegroups.com
I think you are too quick here:

"It seems the subjective interpretation can be ruled out from the start. Logic may be just
subjective, being no more than the inherent structure of human thought. But as such it
cannot declare the necessity of existence. It is absurd and indeed circular to say that
there must be being since we cannot imagine it otherwise. The circularity of such a
proposal follows from the fact that we ourselves, after all, are part of being, so on this
proposal we exist because we cannot imagine ourselves as not existing. In such a scenario,
then, we would be causa sui, since we would have imagined or thought ourselves into
existence. But this is plainly absurd."

First, we can imagine ourselves as not existing. Second, what you call the "subjective
interpretation" I would take to be the anthropic interpretation: We can only ponder this
question if we exist, so we necessarily find being.

Brent

Bruno Marchal

unread,
Oct 22, 2014, 1:09:53 PM10/22/14
to everyth...@googlegroups.com

On 22 Oct 2014, at 17:33, meekerdb wrote:

> On 10/22/2014 1:33 AM, Peter Sas wrote:
>> Hi guys,
>>
>> Here is a blog piece I wrote about nothing as the ultimate source
>> of being:
>>
>> http://critique-of-pure-interest.blogspot.nl/2014/09/why-is-there-something-rather-than.html
>
> I think you are too quick here:
>
> "It seems the subjective interpretation can be ruled out from the
> start. Logic may be just subjective, being no more than the inherent
> structure of human thought. But as such it cannot declare the
> necessity of existence. It is absurd and indeed circular to say that
> there must be being since we cannot imagine it otherwise. The
> circularity of such a proposal follows from the fact that we
> ourselves, after all, are part of being, so on this proposal we
> exist because we cannot imagine ourselves as not existing. In such a
> scenario, then, we would be causa sui, since we would have imagined
> or thought ourselves into existence. But this is plainly absurd."
>
> First, we can imagine ourselves as not existing.

We can do that in the third person way, like we can imagine ourself to
be in Washington *and* Moscow. But we can't imagine ourselves as non
existing in the first person sense. It is meaningless from that first
person point of view. isn't it? Life, topologically, is an "open set",
because (absolute) dying (if that exists) can't be a life experience.
No Diary can ever contained a genuine, non poetical account, of not
existing.
There is a nice poetical account by Italo Calvino "The Nonexistent
Knight", (Le chevalier inexistant), though.



> Second, what you call the "subjective interpretation" I would take
> to be the anthropic interpretation: We can only ponder this question
> if we exist, so we necessarily find being.

We find consciousness, but that is at least a form of "being", OK. It
is Descartes' cogito. If I understand you correctly. We only learn to
bet on more and It get anthropic, or Turing-Löb-tropic assuming comp,
by computer science, indeed only by arithmetical relations. But that
must be shown equivalent with some Everett-tropic QM.

Physics is a sum on all computations. It has to be sigma_1 complete
(if only to have computers) but it might be more than sigma_1
complete, and in all case, the truth *about* the sigma_1 complete
entities escapes all possible theories. Even "divine theories" like
with oracle for a Pi_142 complete problem.

The mathematical logicians have discovered the abyss between the
arithmetical reality of the structure (N, +, *) and all possible
effective theories, (notably by showing that it incarnates all
possible behaviors of all universal machines).

Bruno


>
> Brent
>
> --
> You received this message because you are subscribed to the Google
> Groups "Everything List" group.
> To unsubscribe from this group and stop receiving emails from it,
> send an email to everything-li...@googlegroups.com.
> To post to this group, send email to everyth...@googlegroups.com.
> Visit this group at http://groups.google.com/group/everything-list.
> For more options, visit https://groups.google.com/d/optout.

http://iridia.ulb.ac.be/~marchal/



LizR

unread,
Oct 22, 2014, 9:55:34 PM10/22/14
to everyth...@googlegroups.com
As long as you're sure there is something, rather than nothing... anyway I will have a look, as usual when I have time.

To unsubscribe from this group and stop receiving emails from it, send an email to everything-list+unsubscribe@googlegroups.com.
To post to this group, send email to everything-list@googlegroups.com.
--
You received this message because you are subscribed to the Google Groups "Everything List" group.
To unsubscribe from this group and stop receiving emails from it, send an email to everything-list+unsubscribe@googlegroups.com.
To post to this group, send email to everything-list@googlegroups.com.

LizR

unread,
Oct 22, 2014, 11:34:05 PM10/22/14
to everyth...@googlegroups.com
IMHO this slightly understates the problem of an infinite causal chain:
The idea of an eternally existing universe – for example in the form of an eternal cycle of Big Bangs – might turn out to be a scientifically legitimate hypothesis. It might even turn out to be true. But it still doesn't answer the question why there is anything at all. It doesn't answer the question why there is this infinite series to begin with. It might be objected that this question makes no sense because in an infinite series of causes there simply is no first cause. But this objection assumes that the ultimate cause of the universe must be temporal, existing in time, like the universe itself. But why can't the ultimate cause be non-temporal? This, indeed, is what contemporary physics suggests about the cause of the Big Bang: since not only space and matter but also time itself only came into existence with the Big Bang, the cause of the Big Bang must be timeless. This notion of a non-temporal cause is also inescapable for the infinitist solution. A temporally infinite series of causes has no first cause in time, but it must have an ultimate cause outside of time, a non-temporal cause.
Assuming the laws of physics allow such an infinite chain to exist, I think a more important question is where do those laws originate?

That is, the "something" that we're wondering about includes whatever makes what physically exists the way it is.
 

LizR

unread,
Oct 22, 2014, 11:34:39 PM10/22/14
to everyth...@googlegroups.com
PS apologies if you get onto that later.

Peter Sas

unread,
Oct 23, 2014, 3:50:03 AM10/23/14
to everyth...@googlegroups.com
Well, I'm not a physicists but a philosopher, so I cannot give a physicist's answer. My approach is to start with the most fundamental question (Why is there anything at all?) and then see how far we can get with pure logic alone. It is of course very, very tricky to try to derive fundamental laws of nature in this way. But I think that we can actually get quite far with such an a priori method. Now with respect to your question, I understand that dark energy is a basically repulsive force driving inflation. I don't want to say I can derive dark energy from a priori principles (that would be absurd). But I think I can derive a duality of attraction and repulsion in that way. The reasoning I emply, however, is very abstract, using ideas taken from philosophers like Hegel and Heidegger, although on the whole I feel more attracted to the rationality of Anglo-American philosophy (and science) than to postmodern philosophy (which I think is basically a fraud). Perhaps my reasoning is closest to German idealists like Hegel and Schelling who still feld they could derive the basic principles of natural science from philosophical principles. So here is how my argument goes in nuce, I hope you can make sense of it:

First I argue that nothing is self-negating (for logical arguments see the blog piece). Simply put: nothing is nothing to such a degree that it isn't even itself! Thus, as nothing negates itself, it produces being, it becomes something. Now, since nothing is different from itself, being (as the negation of nothing) must be different from something else. This then is how I define being: as difference from something else. Now it is easy to see that this difference must take two forms. First, being is being because it differs from non-being or nothing (let's call this ontological difference, following Heidegger). Second, being must also be internally differentiated, that is to say: there must be multiple beings differing from each other (let's call this ontic difference). Then we can say: a being is what it is because of its ontic difference from other beings. (Ultimately, I think, this imlies that beings are mathematical, for lacking intrinsic qualities of their own, they canly be distinguished in quantitative ways, such that it is their position in a quantitative structure which determines what they are.) Now we can say: the source (or cause) of what beings are is (ontic) difference. This difference, then, must precede them, just as any origin must precede the originated (at least logically, if not temporally). But what is this difference that precedes the different beings? It's like a relation that generates its own relata. Thus we must postulate something like a pure difference or a pure negativity underlying the mutual non-identity of beings. But what is this pure negativity? It seems clear to me that we are now back with our starting point, the concept of nothing as differing from itself. And this is not surprising if the self-negating nothing generates all beings, for then it must also act as the pure negativity that differentiates beings. But now comes the rub: there is a contradiction between ontological and ontic difference. Recall: ontological difference requires that beings differ from nothing (i.e. pure negativity), whereas ontic difference requires that there is pure negativity between them. Hence: to have existence (i.e. ontological difference) beings must stand in a negative relation to the negativity between them, they must differ from their mutual difference. But to differ from their mutual difference, beings must become the same and loose their separate identities. Hence there is a contradiction between identity and existence, i.e. between the determinacy of beings (ontic difference) and their existence (ontological difference): in short, existence is unifying, determinacy is separating. Now given the fact that being must be logically consistent, we must interpret this contradiction not as logical but as an opposition of forces. Thus existence becomes a unifying force, determinacy (ontic difference) becomes a separating force. The separating force must manifest itself as repulsion, i.e. as resistance against unification. The unifying force must manifest itself as resistance against repulsion, i.e. as attraction. Hence repulsion and attraction are the basic forces that govern being.    

I spelled out this argument in more detail on another blog piece I wrote: http://critique-of-pure-interest.blogspot.nl/2014/06/theses-towards-dialectical-ontology_8246.html
So if you want more detail, please check this piece. I have to emphasize, however, that I am still working on these ideas and that I hope to publish a fuller account on my blog in the near future.





Op woensdag 22 oktober 2014 15:46:16 UTC+2 schreef yanniru:

Richard Ruquist

unread,
Oct 23, 2014, 10:56:19 AM10/23/14
to everyth...@googlegroups.com
On Thu, Oct 23, 2014 at 3:50 AM, Peter Sas <peterj...@gmail.com> wrote:
Well, I'm not a physicists but a philosopher, so I cannot give a physicist's answer. My approach is to start with the most fundamental question (Why is there anything at all?) and then see how far we can get with pure logic alone. It is of course very, very tricky to try to derive fundamental laws of nature in this way. But I think that we can actually get quite far with such an a priori method. Now with respect to your question, I understand that dark energy is a basically repulsive force driving inflation. I don't want to say I can derive dark energy from a priori principles (that would be absurd). But I think I can derive a duality of attraction and repulsion in that way. The reasoning I emply, however, is very abstract, using ideas taken from philosophers like Hegel and Heidegger, although on the whole I feel more attracted to the rationality of Anglo-American philosophy (and science) than to postmodern philosophy (which I think is basically a fraud). Perhaps my reasoning is closest to German idealists like Hegel and Schelling who still feld they could derive the basic principles of natural science from philosophical principles. So here is how my argument goes in nuce, I hope you can make sense of it:

First I argue that nothing is self-negating (for logical arguments see the blog piece). Simply put: nothing is nothing to such a degree that it isn't even itself! Thus, as nothing negates itself, it produces being, it becomes something. Now, since nothing is different from itself, being (as the negation of nothing) must be different from something else. This then is how I define being: as difference from something else. Now it is easy to see that this difference must take two forms. First, being is being because it differs from non-being or nothing (let's call this ontological difference, following Heidegger). Second, being must also be internally differentiated, that is to say: there must be multiple beings differing from each other (let's call this ontic difference). Then we can say: a being is what it is because of its ontic difference from other beings. (Ultimately, I think, this imlies that beings are mathematical, for lacking intrinsic qualities of their own, they canly be distinguished in quantitative ways, such that it is their position in a quantitative structure which determines what they are.) Now we can say: the source (or cause) of what beings are is (ontic) difference. This difference, then, must precede them, just as any origin must precede the originated (at least logically, if not temporally). But what is this difference that precedes the different beings? It's like a relation that generates its own relata. Thus we must postulate something like a pure difference or a pure negativity underlying the mutual non-identity of beings. But what is this pure negativity? It seems clear to me that we are now back with our starting point, the concept of nothing as differing from itself. And this is not surprising if the self-negating nothing generates all beings, for then it must also act as the pure negativity that differentiates beings. But now comes the rub: there is a contradiction between ontological and ontic difference. Recall: ontological difference requires that beings differ from nothing (i.e. pure negativity), whereas ontic difference requires that there is pure negativity between them. Hence: to have existence (i.e. ontological difference) beings must stand in a negative relation to the negativity between them, they must differ from their mutual difference. But to differ from their mutual difference, beings must become the same and loose their separate identities. Hence there is a contradiction between identity and existence, i.e. between the determinacy of beings (ontic difference) and their existence (ontological difference): in short, existence is unifying, determinacy is separating. Now given the fact that being must be logically consistent, we must interpret this contradiction not as logical but as an opposition of forces. Thus existence becomes a unifying force, determinacy (ontic difference) becomes a separating force. The separating force must manifest itself as repulsion, i.e. as resistance against unification. The unifying force must manifest itself as resistance against repulsion, i.e. as attraction. Hence repulsion and attraction are the basic forces that govern being.    

I spelled out this argument in more detail on another blog piece I wrote:

So if you want more detail, please check this piece. I have to emphasize, however, that I am still working on these ideas and that I hope to publish a fuller account on my blog in the near future.
Actually Peter I was thinking more about  your basic assumption that mass-energy is balanced by gravity, one being the negative of the other, which also seems to apply to the dialectic explained in the second blog above, which I just read.

Dark energy creates more space and perhaps spacetime. Space or spacetime does not appear to be the negative of anything. Rather like a particle and an anti-particle annihilating each other to produce light, if the dielectic is correct for Dark Energy, then there must be a balance of positive and negative to create space. Yet the creation of space just creates more Dark Energy along with it. 

The leading candidate for the explanation of Dark Energy is the cosmological constant which amounts to a repulsive force. But space or space time is neutral with respect to force and there is apparently no evidence that an attractive force like gravity due to matter creation is happening. Someone on this list like Brent or John Clark will surely correct my explanation if it is wrong. But in short, Dark Energy appears to falsify the notion that something is derived from nothing by a balance of forces.

BTW there some preliminary evidence that the cosmological constant explanation is not correct: http://www.scientificamerican.com/article/dark-energy-cosmological-constant/
Richard

Bruno Marchal

unread,
Oct 23, 2014, 11:11:32 AM10/23/14
to everyth...@googlegroups.com
Do you agree that computationalism provides an answer, in the sense that it explains why

1) there is no primitive universe  (so no origin problem), 
2) universal machine cannot avoid the discovery of a physical universe and easily take it as primitive,
3) that such physical reality has a first person plural (locally it looks like third person physical) and pure first person non sharable aspects (so this explains, in some sense, the existence of the qualia, and the quanta appears to be sharable one among groups of universal machines).
4) all this in a testable way. In particular, the physics would not be quantum like, computationalism would be no more plausible. Both Gödel's incompleteness theorem and QM saves computationalism and its solution of the mind body problem. 

I m not sure at all such a naïve classical theory is true, but the point is that it is testable, and it gives the most we can hope, because of

5) Logicians can already explains why we cannot explains where the numbers (or the base universal machine you prefer) comes from. We have to assume it to even address the question. Almost by definition, anything capable of explaining the existence of a universal machine, is Turing universal itself. 

This reduces the mind-body problem to our belief in elementary arithmetic, together with an explanation why the question "where elementary arithmetic comes from?" provably unanswerable.

This seems to me to explain why here is something (theological, physical, psychological, biological) from what we accept in high school algebra, up to the possible refutation by nature.

Arithmetic (not logic per se) can be seen then as an atemporal "cause", but from inside, machine needs to bet on larger and larger part of the arithmetical and the analytical truth. The view from "inside" is inexhaustible, and refutes all effective theories. 

Hmm.. To get all this needs some amount of computer science and mathematical logic, and "philosophy of mind".

Bruno

Bruno



--
You received this message because you are subscribed to the Google Groups "Everything List" group.
To unsubscribe from this group and stop receiving emails from it, send an email to everything-li...@googlegroups.com.
To post to this group, send email to everyth...@googlegroups.com.

Peter Sas

unread,
Oct 23, 2014, 2:42:59 PM10/23/14
to everyth...@googlegroups.com
Hi Richard,

I must stress that this is all new territory for me, but what I gather from the things I've read so far is that dark energy is a form of positive energy balanced by the negative energy of gravity. So here too some kind of polarity seems to hold. The point is that as space expands, dark energy increases, so the law of energy conservation is violated, unless the negative energy of gravity increases with an equal amount, so as to 'balance the books'. Thus it would seem that what happens in inflation is precisely this 'splitting' of positive, dark energy (repulsion, which drives the inflation) and the negative energy of gravity. Dark energy can only increase if the negative energy of gravity increases at the same time. Dark energy and gravity seem to be the two opposed sidess of one and the same coin, which is also suggested by the fact that dark energy is often referred to a repulsive gravity.

Well, that's what I understand about it (and that's not much)... I have to do more reading on this subject to feel really comfortable about it... But as far as I can tell right now, this duality of dark energy and the negative energy of gravity fits the dialectical picture of nothing splitting into opposites quite well.

Peter

meekerdb

unread,
Oct 23, 2014, 5:05:42 PM10/23/14
to everyth...@googlegroups.com
On 10/23/2014 12:50 AM, Peter Sas wrote:
Well, I'm not a physicists but a philosopher, so I cannot give a physicist's answer. My approach is to start with the most fundamental question (Why is there anything at all?) and then see how far we can get with pure logic alone. It is of course very, very tricky to try to derive fundamental laws of nature in this way. But I think that we can actually get quite far with such an a priori method. Now with respect to your question, I understand that dark energy is a basically repulsive force driving inflation. I don't want to say I can derive dark energy from a priori principles (that would be absurd). But I think I can derive a duality of attraction and repulsion in that way. The reasoning I emply, however, is very abstract, using ideas taken from philosophers like Hegel and Heidegger, although on the whole I feel more attracted to the rationality of Anglo-American philosophy (and science) than to postmodern philosophy (which I think is basically a fraud). Perhaps my reasoning is closest to German idealists like Hegel and Schelling who still feld they could derive the basic principles of natural science from philosophical principles. So here is how my argument goes in nuce, I hope you can make sense of it:

First I argue that nothing is self-negating (for logical arguments see the blog piece). Simply put: nothing is nothing to such a degree that it isn't even itself! Thus, as nothing negates itself, it produces being, it becomes something.

I like that.  It' just what my friend Norm Levitt used to say, paraphrasing Quine:

"What is there?  Everything! So what isn't there?  Nothing!"



Now, since nothing is different from itself, being (as the negation of nothing) must be different from something else. This then is how I define being: as difference from something else. Now it is easy to see that this difference must take two forms. First, being is being because it differs from non-being or nothing (let's call this ontological difference, following Heidegger). Second, being must also be internally differentiated, that is to say: there must be multiple beings differing from each other (let's call this ontic difference). Then we can say: a being is what it is because of its ontic difference from other beings. (Ultimately, I think, this imlies that beings are mathematical, for lacking intrinsic qualities of their own,

Why must they lack intrinsic qualities?  It seems you are assuming that ontological difference implies multiplicity and the integers.  But what if the something is the universal wave function (as it is in some theories) which is defined over a complex Hilbert space?  Parts are differentiated by their relation to other parts, but there is no definitive way to divide it up.  Rovelli characterizes it as relations without relata.


they canly be distinguished in quantitative ways, such that it is their position in a quantitative structure which determines what they are.) Now we can say: the source (or cause) of what beings are is (ontic) difference. This difference, then, must precede them, just as any origin must precede the originated (at least logically, if not temporally). But what is this difference that precedes the different beings? It's like a relation that generates its own relata. Thus we must postulate something like a pure difference or a pure negativity underlying the mutual non-identity of beings. But what is this pure negativity? It seems clear to me that we are now back with our starting point, the concept of nothing as differing from itself. And this is not surprising if the self-negating nothing generates all beings, for then it must also act as the pure negativity that differentiates beings. But now comes the rub: there is a contradiction between ontological and ontic difference. Recall: ontological difference requires that beings differ from nothing (i.e. pure negativity), whereas ontic difference requires that there is pure negativity between them. Hence: to have existence (i.e. ontological difference) beings must stand in a negative relation to the negativity between them, they must differ from their mutual difference. But to differ from their mutual difference, beings must become the same and loose their separate identities.

I don't see how that follows.  You and I both differ from Bruno, but that doesn't entail that you can I are the same person.


Hence there is a contradiction between identity and existence, i.e. between the determinacy of beings (ontic difference) and their existence (ontological difference): in short, existence is unifying, determinacy is separating. Now given the fact that being must be logically consistent, we must interpret this contradiction not as logical but as an opposition of forces.

"Forces" opposing one another (are we to assume metaphysical equilibrium) seems more metaphorical than logical. 

Brent

Bruce Kellett

unread,
Oct 23, 2014, 6:40:22 PM10/23/14
to everyth...@googlegroups.com
Peter Sas wrote:
> Hi Richard,
>
> I must stress that this is all new territory for me, but what I gather
> from the things I've read so far is that dark energy is a form of
> positive energy balanced by the negative energy of gravity. So here too
> some kind of polarity seems to hold. The point is that as space expands,
> dark energy increases, so the law of energy conservation is violated,
> unless the negative energy of gravity increases with an equal amount, so
> as to 'balance the books'. Thus it would seem that what happens in
> inflation is precisely this 'splitting' of positive, dark energy
> (repulsion, which drives the inflation) and the negative energy of
> gravity. Dark energy can only increase if the negative energy of gravity
> increases at the same time. Dark energy and gravity seem to be the two
> opposed sidess of one and the same coin, which is also suggested by the
> fact that dark energy is often referred to a repulsive gravity.


The idea that the positive mass-energy of the universe is balanced by
the negative energy of gravitation is quite common in the popular
science literature -- the idea is that one can then get zero total
energy and explain a universe coming from nothing.

The trouble with this idea is that it is flatly contradicted by general
relativity. There are two main points here. First, in the cosmological
models of GR, energy is not generally conserved. Energy conservation on
the large scale depends on the existence of a time-like Killing vector
field, and no such field exists in the general non-static spacetime,
such as an expanding universe. The question of the total energy of the
universe simply has no answer -- no such total energy can be defined so
it has no value -- zero or anything else.

The second point is that GR is based on the idea that energy, of
whatever form, is a source term for gravity. The equations of GR have
the geometry of spacetime depending solely on the stress-energy tensor
containing all mass, energy, stress, pressure and other physical terms.
There is no term for negative gravitational energy in this tensor.
Negative gravitational energy does not affect the geodesics of the
spacetime, it does not affect the orbits of distant satellites, for
instance. So, in a very real sense, it does not exist. It can be
described only by what is commonly called a pseudo-tensor. That is, a
quantity that does not transform as a tensor under coordinate
transformations. One can always find a frame in which so-called negative
gravitational energy vanishes, so it is not physical.

Hope this helps clear up a few confusions.

Bruce
> <http://www.scientificamerican.com/article/dark-energy-cosmological-constant/>
> Richard

Peter Sas

unread,
Oct 24, 2014, 1:35:51 PM10/24/14
to everyth...@googlegroups.com
Hi Brent,

On my account, beings (i.e. all things that are) lack intrinsic qualities because they are defined through their differences from each other. Thus a being is what it is simply by not being something else. So in themselves, abstracted from their relations to other beings, beings 'are' just nothing, indeterminate, hence they lack intrinsic qualities (all properties are relational). If you like you can also say there are just relations and not relata, or alternatively that there are only internal relations of which the relata are functions. The next question would then be: but what kind of relations are ontologically most basic? I would say: mathematical relations.

According to me, saying that a being is what it is because it differs from something else is the same as saying that all being is mathematical. For if beings lack all intrinsic qualities, they can only be distinguished quantitatively, and that's basically what mathematics is about, isn't it? This seems to me to be the reason why the whole of mathematics (and everything that can be described mathematically) can ultimately be described in binary terms, as compositions of the difference between 1 and 0, which is just difference as such. It seems to me that mathematics is what you get when you take a structuralist view of things, where you say that a thing IS just its differences from something else.

I think this also the view Tegmark takes in his Mathematical Universe book, although he speaks of "relation" instead of "difference":

"the only properties of these entities would be those embodied by the relations between them... To a modern logician, a mathematical structure is precisely this: a set of abstract entities with relations between them. Take the integers, for instance, [...] the only properties of integers are those embodied by the relations between them." (p.259) "5 has the property that it's the sum of 4 and 1, say, but it's not yellow, and it's not made of anything." (p.268) " the entities of a mathematical structure are purely abstract, which means that they have no intrinsic properties whatsoever..." (p.264)

So what is primary, "relation" or "difference"? I would say neither, both terms seem equally primordial. For to be able to specify which relations hold between, say, 5 and 4, you first have to specify how they differ from each other, e.g. by saying 5 = 4 +1. But that's the same as saying what relations hold between these entities? Thus it would seem that mathematical relations are just relations of difference, indeed, ultimately the 'pure', binary difference of 1 and 0.

According to me, such a mathematical view of being as defined by difference (ultimately 1 and 0) follows from reflection on nothing. Nothing is inconsistent, hence it differs from itself. Being then is the (self-)negation of nothing, hence it must be difference (not from itself but) from something else. This then is what "being" means: to differ from something else, and as we have just seen this is just what mathematics is about.

As for the fact that you and I both differ from Bruno but we are obviously not the same, that's because you and I differ as well... If you like, in terms of the above account, you could say Bruno, you and me are all qualititatively indistinguishable units which nevertheless have different values only because of our different positions in a quantitative structure, e.g. spacetime.    

Spinoza famously said, and Hegel repeated it: every determination is a negation, i.e. saying what something is is saying what it is not, i.e. a thing IS its difference form something else. You can call this dialectics, but according to me it converges with a mathematical view of being, so to that extent there is a strong dialectical aspect to mathematics. Or at least so it seems to me.

Then another question arises, which relates to the topic of consciousness and how it fits in the physical (= mathematical) world. If all beings are ultimately mathematical (quantitative) in nature, then where does quality come from? It would seem to me that quality is precisely an intrinsic property, which does not depend on relation to something else. Take as a thought experiment someone who right after birth was given red colored glasses so that everything looks red to him, and he has been wearing the glasses all of his life, so he has never seen any other color, all he sees are shades of red. Obviously, then, it must be possible to be aware of red without being aware of other colors. Hence it is an intrinsic quality. In contrast, you cannot be aware of 1 without being aware of other numbers, for knowing what "1" means is simply knowing that 1 is more than 0, that 1+1=2 etc. Ultimately, then, I think we can pose the problem of consciousness in terms of the quantity/quality opposition. If reality is ultimately mathematical (quantitative), how then are qualities possible?

Peter Sas

unread,
Oct 24, 2014, 1:36:40 PM10/24/14
to everyth...@googlegroups.com
Hi Bruce,

Thanks for your explanation, but I'm afraid it doesn't really help me. The main reason is no doubt my own stupidity, since most of what you say goes over my head. I understand some physics, but it must be explained to me in non-mathematical terms, otherwise I don't get (I have asperger's AND dyscalculia, so go figure...). I'm good at abstract reasoning, but very bad at formulae and numbers, since I mix them up... unfortunately, since I'm totally fascinated by math in a philosophical way.

So all I know about physics is from reading pop science books like Hawking, Gribbin, Brian Green etc. So who then must I believe? I read Hawking saying that the positive energy of mass is balanced by the negative energy of gravity so the net amount of energy in the universe is zero. And now you tell me that's not true!

So what gives? I wish you physicists would make up your mind ;)

Perhaps it has to do with the fact that most physicists just calculate (not that there is anything wrong with that, of course, math is the key to modern science). But when it comes to explaining what these calculations mean, things get tricky, and you find physicists claiming different things. Perhaps this (positive energy vs. negative energy) could be one of those things?

Peter

Bruce Kellett

unread,
Oct 24, 2014, 6:12:00 PM10/24/14
to everyth...@googlegroups.com
Peter Sas wrote:
> Hi Bruce,
>
> Thanks for your explanation, but I'm afraid it doesn't really help me.
> The main reason is no doubt my own stupidity, since most of what you say
> goes over my head. I understand some physics, but it must be explained
> to me in non-mathematical terms, otherwise I don't get (I have
> asperger's AND dyscalculia, so go figure...). I'm good at abstract
> reasoning, but very bad at formulae and numbers, since I mix them up...
> unfortunately, since I'm totally fascinated by math in a philosophical way.
>
> So all I know about physics is from reading pop science books like
> Hawking, Gribbin, Brian Green etc. So who then must I believe? I read
> Hawking saying that the positive energy of mass is balanced by the
> negative energy of gravity so the net amount of energy in the universe
> is zero. And now you tell me that's not true!

Unfortunately, physics is a subject where the text books tend to carry
more weight than the popular presentations. The text books show that the
claims about the zero net energy of the universe made by people such as
Hawking and Krauss in popular presentations are wrong. The interesting
question is why undoubtedly clever people such as Krauss and Hawking
would make such fallacious claims. I suppose simplification can
sometimes be indistinguishable from over-simplification -- or else
people become more susceptible to brain farts as they get older.....

Bruce

Peter Sas

unread,
Oct 25, 2014, 12:23:21 AM10/25/14
to everyth...@googlegroups.com
Wow... That's quite shocking! I see I have to be much more careful in taking over what the pop science writers say... 

Terren Suydam

unread,
Oct 25, 2014, 10:51:35 AM10/25/14
to everyth...@googlegroups.com

I find this quite surprising too and wonder if Brent could weigh in as I'm out of my league on that stuff.

Terren

--

meekerdb

unread,
Oct 25, 2014, 12:01:13 PM10/25/14
to everyth...@googlegroups.com
Bruce is a very good physicist and he's right.  John Baez has a good discussion of the point on his blog.

Brent

spudb...@aol.com

unread,
Oct 25, 2014, 9:02:25 PM10/25/14
to everyth...@googlegroups.com


Sent from AOL Mobile Mail

 I like Larry Krauss despite his attacks on Frank Tipler, because Larry Krauss also concedes the possibility of faster than light travel. No which among us, are going to turn down Star Trek?

Bruce Kellett

unread,
Oct 26, 2014, 1:35:27 AM10/26/14
to everyth...@googlegroups.com
I am with you that generally Krauss does a good job of popularizations
of cosmology and so on. He is generally quite careful and accurate in
his book "A Universe from Nothing", except on page 166, where he says
"There is one universe in which the total energy is definitely and
precisely zero.... It is a closed universe..." This is just simply
incorrect. The total mass energy of a closed universe is not definable
because there is no reference point outside such a universe from which
one can measure the total enclosed energy. Krauss's argument by analogy
with the total charge in the universe fails because he appears to have
overlooked the simple fact that in a closed universe, light cannot go
right round and back to the starting point before the universe
re-contracts to zero size. This is a simple GR calculation in the
geometry of a closed universe. See the text by Misner, Thorne and
Wheeler (MTW, the 'Bible' of general relativists!)

Bruce




spudboy100 via Everything List wrote:
>
> Sent from AOL Mobile Mail
>
> I like Larry Krauss despite his attacks on Frank Tipler, because Larry
> Krauss also concedes the possibility of faster than light travel. No
> which among us, are going to turn down Star Trek?
>
> From: meekerdb <meek...@verizon.net>
>
> Bruce is a very good physicist and he's right. John Baez has a good
> discussion of the point on his blog.
>
> Brent
>
> On 10/25/2014 7:51 AM, Terren Suydam wrote:
>
> I find this quite surprising too and wonder if Brent could weigh in
> as I'm out of my league on that stuff.
>
> Terren
>
> On Oct 25, 2014 12:23 AM, "Peter Sas" < peterj...@gmail.com

Bruno Marchal

unread,
Oct 26, 2014, 10:13:00 AM10/26/14
to everyth...@googlegroups.com
On 24 Oct 2014, at 19:35, Peter Sas wrote:

Hi Brent,

On my account, beings (i.e. all things that are) lack intrinsic qualities because they are defined through their differences from each other.

I guess you love category theory, which is mathematics based on that idea. It is also a quite functionalist and sort of constructivist view, like an employee will be defined by its job, and not by the particular individual having that job.

It works very well for many branch of math, but it is in trouble for computer science, and some other branch of logic.
Some mathematical object can have intrinsic quality. Modal logic is a good tool for handling this.

Note also that a "universe" is usually considered only for its intrinsic quality. A universe has a priori no relation with something else, as everything is or should be part of a universe, by definition.

I could argue that it is the same for a dreamer, of any closed system in which we are interested.




Thus a being is what it is simply by not being something else. So in themselves, abstracted from their relations to other beings, beings 'are' just nothing, indeterminate, hence they lack intrinsic qualities (all properties are relational). If you like you can also say there are just relations and not relata, or alternatively that there are only internal relations of which the relata are functions. The next question would then be: but what kind of relations are ontologically most basic? I would say: mathematical relations.

That follows from the computationalist hypothesis. You can read the references in my URL. Or the posts to this list. If we assume that the brain (or whatever my consciousness supervene on) is Turing emulable, we must recover physics from a special self-referential statistics on the computations. Physics becomes a branch of machine's psychology, or better machine's theology (in the greek original sense of the word) itself branch of arithmetic or mathematics.




According to me, saying that a being is what it is because it differs from something else is the same as saying that all being is mathematical. For if beings lack all intrinsic qualities, they can only be distinguished quantitatively, and that's basically what mathematics is about, isn't it?

You are quick and a bit vague on this.


This seems to me to be the reason why the whole of mathematics (and everything that can be described mathematically) can ultimately be described in binary terms, as compositions of the difference between 1 and 0, which is just difference as such.

I doubt you will get more than the numbers, or than the computable. In fact all attempt to define mathematically the hole of mathematics fails. In fact, already for the arithmetical reality, it follows from Gödel's incompleteness that all axiomatizable theories will fail to unify it. The arithmetical reality is inconceivably large.




It seems to me that mathematics is what you get when you take a structuralist view of things, where you say that a thing IS just its differences from something else.

You get category theory.




I think this also the view Tegmark takes in his Mathematical Universe book, although he speaks of "relation" instead of "difference":

"the only properties of these entities would be those embodied by the relations between them... To a modern logician, a mathematical structure is precisely this: a set of abstract entities with relations between them.

That is more the view of an algebraist. A logicien studies such strcture as model of theories (set of sentences or propositions). The algebraists and categoricians study the relation between structures. The logician study the relation between those relations and theories or machines (syntactical beings).



Take the integers, for instance, [...] the only properties of integers are those embodied by the relations between them." (p.259) "5 has the property that it's the sum of 4 and 1, say, but it's not yellow, and it's not made of anything." (p.268) " the entities of a mathematical structure are purely abstract, which means that they have no intrinsic properties whatsoever..." (p.264)

We can doubt this, notably for the numbers where many particular numbers can be individuated through its special property.




So what is primary, "relation" or "difference"? I would say neither, both terms seem equally primordial. For to be able to specify which relations hold between, say, 5 and 4, you first have to specify how they differ from each other, e.g. by saying 5 = 4 +1. But that's the same as saying what relations hold between these entities? Thus it would seem that mathematical relations are just relations of difference, indeed, ultimately the 'pure', binary difference of 1 and 0.

Brouwer founded mathematics on something similar, but he get the constructive mathematics (and only one of a special kind). Most of the arithmetical reality is bigger than what such subjectivist theories are about. Of course the ability to accept the difference between 0 and 1 is fundamental and very important.




According to me, such a mathematical view of being as defined by difference (ultimately 1 and 0) follows from reflection on nothing. Nothing is inconsistent, hence it differs from itself. Being then is the (self-)negation of nothing, hence it must be difference (not from itself but) from something else. This then is what "being" means: to differ from something else, and as we have just seen this is just what mathematics is about.

As for the fact that you and I both differ from Bruno but we are obviously not the same, that's because you and I differ as well... If you like, in terms of the above account, you could say Bruno, you and me are all qualititatively indistinguishable units which nevertheless have different values only because of our different positions in a quantitative structure, e.g. spacetime.    

Spinoza famously said, and Hegel repeated it: every determination is a negation,

That is interesting, and, I think,  well illustrated by formal provability (which is on the determination part) and acts formally as a negation for many propositions: like []<>t -> []f  (proving self-consistency entails proving a falsity). Also, the Löb rule or axiom shows also that provability is a form of negation. 
In fact even the simple formal implication is also a sort of negation p -> q is really put for NOT p ... or  q. But this is weaker than the illustration just above.




i.e. saying what something is is saying what it is not,

OK, but the reverse is not true.
God and the protegorean virtues are defined only negatively. You can not define them by saying what they are, only by what they are not. God is not this, neither that, nor ... (cf the greek negative theologies).



i.e. a thing IS its difference form something else.

OK, you might say God is what is different from all beings.


You can call this dialectics, but according to me it converges with a mathematical view of being, so to that extent there is a strong dialectical aspect to mathematics. Or at least so it seems to me.

OK.



Then another question arises, which relates to the topic of consciousness and how it fits in the physical (= mathematical) world.

I stringly disagree with making the physical = to the mathematical. The physical might have a mathematical origin, but the mathematical is larger than the physical. mathematical objects are not physical objects. The beliefs in physicc are mathematical object, and they should arise from some mathematical special relation. In fact, to put it crudely, the physical might be the border of the mathematical when seen from internal (to math) beings.




If all beings are ultimately mathematical (quantitative)

I would also be careful not to identify math and the study of the quantitative. In computer science qualities arise from the intensional part of arithmetic, where an object can refer to its own code, and see the difference with truth to which he has some access, without being able to capture them in pure third person relations.



in nature,

Well, what is that? (Especially if you agree that math is more fundamental than physics).



then where does quality come from? It would seem to me that quality is precisely an intrinsic property, which does not depend on relation to something else. Take as a thought experiment someone who right after birth was given red colored glasses so that everything looks red to him, and he has been wearing the glasses all of his life, so he has never seen any other color, all he sees are shades of red. Obviously, then, it must be possible to be aware of red without being aware of other colors.

I don't see why that would be necessary.


Hence it is an intrinsic quality. In contrast, you cannot be aware of 1 without being aware of other numbers, for knowing what "1" means is simply knowing that 1 is more than 0, that 1+1=2 etc. Ultimately, then, I think we can pose the problem of consciousness in terms of the quantity/quality opposition. If reality is ultimately mathematical (quantitative), how then are qualities possible?

By being some semantical fixed point. It is where the map is confused with the territory, and that happens when the map is part of the territory. What is nice is that such fixed points obeys logics already suggested for the Qualia (and it generalizes a bit quantum logic). See for example the paper by J.L. Bell (the logician, not the physicists).

Bruno




--
You received this message because you are subscribed to the Google Groups "Everything List" group.
To unsubscribe from this group and stop receiving emails from it, send an email to everything-li...@googlegroups.com.
To post to this group, send email to everyth...@googlegroups.com.
Visit this group at http://groups.google.com/group/everything-list.
For more options, visit https://groups.google.com/d/optout.

spudb...@aol.com

unread,
Oct 26, 2014, 11:47:45 AM10/26/14
to everyth...@googlegroups.com
And now in physics we have this-
 
 
MWI worlds interact
 
 
 
-----Original Message-----
From: Bruno Marchal <mar...@ulb.ac.be>
To: everything-list <everyth...@googlegroups.com>
Sent: Sun, Oct 26, 2014 10:13 am
Subject: Re: Why is there something rather than nothing? From quantum theory to dialectics?

Peter Sas

unread,
Oct 26, 2014, 12:52:39 PM10/26/14
to everyth...@googlegroups.com
Hi Brent,

Thanks for your comments, which are very useful, even if the more technical comments are beyond me (I have to study up on that). Thanks for the tip about category theory, I vaguely heard about it... I know it is a rival to set theory when it comes to founding math (insofar that is possible given Goedel).

You write: "Note also that a "universe" is usually considered only for its intrinsic quality. A universe has a priori no relation with something else, as everything is or should be part of a universe, by definition."

I would say: what is outside the universe is precisely nothing, which is why the universe exists in the first place, that is, it is not nothing (= ontological difference). So even for the universe it holds that it is what it is by differing from what it is not. And if it differs from nothing, then it must also be determined (internally differentiated = ontic difference) otherwise it would be indeterminate and thus as good as nothing.


i.e. a thing IS its difference form something else.

OK, you might say God is what is different from all beings.

In that sense I would say: God is really nothing, since it makes all things be by differing from them.

You write: "If we assume that the brain (or whatever my consciousness supervene on) is Turing emulable, we must recover physics from a special self-referential statistics on the computations. Physics becomes a branch of machine's psychology, or better machine's theology (in the greek original sense of the word) itself branch of arithmetic or mathematics."

I don't get this. I see how the brain/consciousness might correspond to self-referential loops in computations, but why does this have implications for the whole of physics? Do you mean to say that there must be a compuational approach to God as the creator of physical nature?

You write: "We can doubt this, notably for the numbers where many particular numbers can be individuated through its special property." Could you give an example? I would say: even for unique numbers (unique primes?) it holds that they are only what they are because of their place in the number system; take the system away and the number is just a meaningless mark.

Peter


spudb...@aol.com

unread,
Oct 26, 2014, 5:25:09 PM10/26/14
to everyth...@googlegroups.com
Yeah Bruce,
Things could be much different then what Krauss conceives of today. Why not a closed timelike curve within an open universe, or an open universe within a CTC?? I always ask myself, how this can benefit our species? If its just a fact that is too big and indifferent to our existence, then I sort of back-burner the scientist and his paper to file 13. I mean, it can be profoundly true and deeply, useless. Being philosophical, as I kind of am, forces me to use Maslow's hierarchy of needs quite frequently, which is why I am no physicist.
 
 
-----Original Message-----
From: Bruce Kellett <bhke...@optusnet.com.au>
To: everything-list <everyth...@googlegroups.com>
Sent: Sun, Oct 26, 2014 1:35 am
Subject: Re: Why is there something rather than nothing? From quantum theory to dialectics?

>> wrote:
> 
>         Wow... That's quite shocking! I see I have to be much more
>         careful in taking over what the pop science writers say... 
> 
>         Unfortunately, physics is a subject where the text books tend to
>         carry
> 
>             more weight than the popular presentations. The text books
>             show that the
>             claims about the zero net energy of the universe made by
>             people such as
>             Hawking and Krauss in popular presentations are wrong. The
>             interesting
>             question is why undoubtedly clever people such as Krauss and
>             Hawking
>             would make such fallacious claims. I suppose simplification can
>             sometimes be indistinguishable from over-simplification --
>             or else
>             people become more susceptible to brain farts as they get
>             older.....
> 
>             Bruce

-- 
You received this message because you are subscribed to the Google Groups 
"Everything List" group.
To unsubscribe from this group and stop receiving emails from it, send an email 
to everything-li...@googlegroups.com.
To post to this group, send email to everyth...@googlegroups.com.
Visit this group at http://groups.google.com/group/everything-list.
For more options, visit https://groups.google.com/d/optout.

LizR

unread,
Oct 26, 2014, 6:32:48 PM10/26/14
to everyth...@googlegroups.com
Mind you as some people like to point out, we know GR is wrong...

John Clark

unread,
Oct 26, 2014, 6:55:34 PM10/26/14
to everyth...@googlegroups.com
On Fri, Oct 24, 2014 at 7:12 PM, Bruce Kellett 

> the claims about the zero net energy of the universe made by people such as Hawking and Krauss in popular presentations are wrong. The interesting question is why undoubtedly clever people such as Krauss and Hawking would make such fallacious claims.

Their error is easily explained,  Hawking and Krauss just aren't as smart as you are. 

   John K Clark
 

John Clark

unread,
Oct 26, 2014, 8:08:35 PM10/26/14
to everyth...@googlegroups.com
On Sun, Oct 26, 2014 at 1:35 AM, Bruce Kellett 
 
> he [Krauss] appears to have overlooked the simple fact that in a closed universe, light cannot go right round and back to the starting point before the universe re-contracts to zero size.

You appear to have overlooked the simple fact that in a universe that is getting smaller and smaller the time it takes to circumnavigate that universe becomes less and less. 

> Krauss's argument by analogy with the total charge in the universe fails 

I don't see why, it seems like a pretty good analogy to me.  Krauss's argument is about electric field lines joining up and a electric field moves at exactly the same as light does, so if it works for one it should work for both. 

  John K Clark

 

spudb...@aol.com

unread,
Oct 26, 2014, 10:09:55 PM10/26/14
to everyth...@googlegroups.com


Sent from AOL Mobile Mail
 Since when is general relativity, wrong? What news did I miss?



-----Original Message-----
From: LizR <liz...@gmail.com>
To: everything-list <everyth...@googlegroups.com>
Sent: Sun, Oct 26, 2014 05:32 PM
Subject: Re: Why is there something rather than nothing? From quantum theory to dialectics?


Mind you as some people like to point out, we know GR is wrong...

spudb...@aol.com

unread,
Oct 26, 2014, 10:14:48 PM10/26/14
to everyth...@googlegroups.com


Sent from AOL Mobile Mail

 Pure serious great from a theoretical physicist point of view until their series get disapproved by improved equipment. So, vast, and improved sky survey, might prove or disprove countless theories. All of the series and interpretations of theoretical physicist however, seem to have been proved right by many tests. For instance, physicists, Todd Brunn, who has worked on the notion of Closed Timelike Curves, and apply them to computer systems, seems to have born some fruit. Moreover, it seems likely to have cosmological applications, and explaining our universe, as opposed to Jimmy Joe Bob's universe which is just down the street. For me a likely better question to ask is what is our goal, through science, and through philosophy? Or, more concisely, where do you want us to be? 



-----Original Message-----
From: John Clark <johnk...@gmail.com>
To: everything-list <everyth...@googlegroups.com>
Sent: Sun, Oct 26, 2014 05:55 PM
Subject: Re: Why is there something rather than nothing? From quantum theory to dialectics?


--

Bruce Kellett

unread,
Oct 26, 2014, 10:15:15 PM10/26/14
to everyth...@googlegroups.com
John Clark wrote:
> On Sun, Oct 26, 2014 at 1:35 AM, Bruce Kellett
>
> > he [Krauss] appears to have overlooked the simple fact that in a
> closed universe, light cannot go right round and back to the
> starting point before the universe re-contracts to zero size.
>
> You appear to have overlooked the simple fact that in a universe that is
> getting smaller and smaller the time it takes to circumnavigate that
> universe becomes less and less.

That is true, but it is relatively simple to calculate the time for a
light beam to go round a closed universe and also the lifetime of such a
universe before it re-contracts to zero. You find that if the universe
is dominated by radiation, then light can get round exactly once. If it
is dominated by matter, then light can only ever get half way round. If
there is more that a very small amount of dark energy, then a beam of
light can never get right round the universe (the universe does not
re-contract in that case -- it expands for ever even though closed). So
you can never see the back of your own head.

The calculations are in Misner, Thorne and Wheeler (MTW).

Bruce

John Clark

unread,
Oct 26, 2014, 11:40:12 PM10/26/14
to everyth...@googlegroups.com
On Sun, Oct 26, 2014  Bruce Kellett <bhke...@optusnet.com.au> wrote:

>If there is more that a very small amount of dark energy, then a beam of light can never get right round the universe (the universe does not re-contract in that case

OK.
 
>it expands for ever even though closed). So you can never see the back of your own head.

Obviously if it expands forever you could never see the back of your head, and our universe is not only expanding its accelerating; but how could you call such a universe closed?

  John K Clark
   
  

Bruce Kellett

unread,
Oct 26, 2014, 11:47:44 PM10/26/14
to everyth...@googlegroups.com
It has a finite extent, even if expanding exponentially. The point is
that the intrinsic curvature is positive in the case under discussion,
which is what we mean by a closed universe.

Bruce

>
> John K Clark

LizR

unread,
Oct 27, 2014, 6:09:56 AM10/27/14
to everyth...@googlegroups.com
On 27 October 2014 15:09, spudboy100 via Everything List <everyth...@googlegroups.com> wrote:

Sent from AOL Mobile Mail
 Since when is general relativity, wrong? What news did I miss?

It's (generally :-) assumed to break down in the vicinity of (what would otherwise be) singularities.

spudb...@aol.com

unread,
Oct 27, 2014, 9:16:45 AM10/27/14
to everyth...@googlegroups.com
Ok Hawkings old sales pitch! Thanks.
It's (generally :-) assumed
 
 
 
-----Original Message-----
From: LizR <liz...@gmail.com>
To: everything-list <everyth...@googlegroups.com>
Sent: Mon, Oct 27, 2014 6:09 am
Subject: Re: Why is there something rather than nothing? From quantum theory to dialectics?

Bruno Marchal

unread,
Oct 27, 2014, 10:48:13 AM10/27/14
to everyth...@googlegroups.com
On 26 Oct 2014, at 16:47, spudboy100 via Everything List wrote:

And now in physics we have this-
 
 
MWI worlds interact


Then QM is wrong. Weinberg but also Plaga (on this list) showed that [QM is non linear, but approximatively correct] is equivament with QM worlds can interact.

Bruno

Bruno Marchal

unread,
Oct 27, 2014, 10:52:57 AM10/27/14
to everyth...@googlegroups.com
On 27 Oct 2014, at 03:09, spudboy100 via Everything List wrote:



Sent from AOL Mobile Mail
 Since when is general relativity, wrong? What news did I miss?

Quantum Mechanics. I guess. But GR per se seems not consistent by itself. A bit like Newton physics, it seems it divides by zero, and Wheeler coined the term "black hole" for this happenings, which were a name for the contradiction. But QM saves the black-hole, apparently, but still refuse a simple marriage with GR. Most physicists, I think, believe that QM is probably less faulty than GR.

Bruno





-----Original Message-----
From: LizR <liz...@gmail.com>
To: everything-list <everyth...@googlegroups.com>
Sent: Sun, Oct 26, 2014 05:32 PM
Subject: Re: Why is there something rather than nothing? From quantum theory to dialectics?


Mind you as some people like to point out, we know GR is wrong...


--
You received this message because you are subscribed to the Google Groups "Everything List" group.
To unsubscribe from this group and stop receiving emails from it, send an email to everything-li...@googlegroups.com.
To post to this group, send email to everyth...@googlegroups.com.
Visit this group at http://groups.google.com/group/everything-list.
For more options, visit https://groups.google.com/d/optout.

--
You received this message because you are subscribed to the Google Groups "Everything List" group.
To unsubscribe from this group and stop receiving emails from it, send an email to everything-li...@googlegroups.com.
To post to this group, send email to everyth...@googlegroups.com.
Visit this group at http://groups.google.com/group/everything-list.
For more options, visit https://groups.google.com/d/optout.

Bruno Marchal

unread,
Oct 27, 2014, 11:55:15 AM10/27/14
to everyth...@googlegroups.com
Hi Peter,

You are replying to my post (I am Bruno, not Brent, although I am open that we are all the same person, it is better to keep the name right for helping in future references)


On 26 Oct 2014, at 17:52, Peter Sas wrote:


Thanks for your comments, which are very useful, even if the more technical comments are beyond me (I have to study up on that). Thanks for the tip about category theory, I vaguely heard about it... I know it is a rival to set theory when it comes to founding math (insofar that is possible given Goedel).

You write: "Note also that a "universe" is usually considered only for its intrinsic quality. A universe has a priori no relation with something else, as everything is or should be part of a universe, by definition."

I would say: what is outside the universe is precisely nothing,

That might already be too much. Also, the notion of nothing is theory dependent. The quantum nothingness is not the same as the nothingness in set theory, etc.



which is why the universe exists in the first place, that is, it is not nothing (= ontological difference).

That looks like a play with word, which does not mean that there is not some truth behind, but you will have to elaborate a lot.



So even for the universe it holds that it is what it is by differing from what it is not. And if it differs from nothing, then it must also be determined (internally differentiated = ontic difference) otherwise it would be indeterminate and thus as good as nothing.

Hmm...



i.e. a thing IS its difference form something else.

Not sure I wrote this.



OK, you might say God is what is different from all beings.


But I did say that.




In that sense I would say: God is really nothing, since it makes all things be by differing from them.

That idea is made precise in Plotinus' theory (for which I provide a way to interpret it in arithmetic, with God played by the notion of Arithmetical truth or reality, the Noùs is interpreted by Gödel's provability predicate, and then soul and matter are intensional variants from that predicate, restricted to computationnaly accessible states).

In that case God is responsible for the beings (the natural numbers and their "ideas"), but is not a being itself. In fact in plotinus, and in the machine's classical theology, both God and Matter are outside the universe (the realm of being, the Noùs).




You write: "If we assume that the brain (or whatever my consciousness supervene on) is Turing emulable, we must recover physics from a special self-referential statistics on the computations. Physics becomes a branch of machine's psychology, or better machine's theology (in the greek original sense of the word) itself branch of arithmetic or mathematics."

I wrote that indeed.



I don't get this. I see how the brain/consciousness might correspond to self-referential loops in computations, but why does this have implications for the whole of physics?

That was the object of a life of research, and thus is not that easy to understand without some amount of work. I have not so much time in this period, and so I refer you to my most simple paper, though a bit concise, and the second part needs some familiarity with mathematical logics and theoretical computer science. Nevertheless, the implication for physics does not need special knowledge but passive computer science:


I assume Mechanism, actually its digital version, which makes a lot of sense thanks to the Church (Post-Turing-Kleene) thesis. It forces to generalise Everett embedding of the physicists and engineers into the physical realm, into an embedding of the mathematicians and dreamers into the arithmetical realm. The quantum wave should emerge phenomenologically from the statistical "interference" of all computations, independently of any languages describing the computation. If digital mechanism, alias computationalism, is correct we get a many-dreams interpretation of arithmetic, and the physical realities (or reality) percolate(s) from coherence conditions that some computations have. But we have to derive them constructively to verify that statement, and the math of self-reference has made possible to derive the arithmetical quantization, and up to now, it is enough quantum-like to say that computationalism is not (yet) refuted.

It is my main point: that some problem in philosophy and theology can be approached with the scientific method, notably when using the computationalist hypothesis, which makes the ideally correct case amenable to pure mathematics.



Do you mean to say that there must be a compuational approach to God as the creator of physical nature?

Not really. I say that it has to be like that, if we assume that our brain (quantum or classical) are Turing emulable.

Then God appears to be highly non computable (in fact not even definable by the machine) as it is the whole arithmetical truth a highly non computable set, it is at the top of the arithmetical hierarchy which classifies the degrees of non-computability or non-solvability).  Then it does not ring so well to see that God as a creator, it looks more like the TAO, or the ONE of Plotinus. The arithmetical reality implements by itself all computations, but also many levels of truth about those computations. The difference which play the important role in all of this, is the difference between what the machine will be able to justify (in different senses) and what she can conceive or imagine, and with reality/truth. 




You write: "We can doubt this, notably for the numbers where many particular numbers can be individuated through its special property." Could you give an example? I would say: even for unique numbers (unique primes?) it holds that they are only what they are because of their place in the number system; take the system away and the number is just a meaningless mark.

I can agree with that. But the same reasoning in quantum field theory would lead that only the vacuum exist. But in science we can adopt the semi-axiomatic approach, and never try to say what things are, but trying only to agree on some axioms about them. So I am OK with what you say, but if only by methodology, I apply this on all things. 

Now, some mathematical object are intended to instanciate many different things, like the theory of groups (there are many different groups). Other theories, like number theory, or machine's theory, talk about only one structure (the natural numbers). We can't define completely the structure N, with only the means in N, we need sets to do that, or second logical axioms, so like groups, there are many "natural numbers" systems. Unlike groups, we can focus on the standard N structure, and the non standard play a bit a role of reservoir of simplification tools, like analysis.

Eventually all this reduce the mind-body problem, and the reason why there is nothing, to the mystery of our understanding and beliefs about arithmetic, and this provides a tool for explaining why machines/number can't sole that question. So, computationalism seems to offer the best we can hope: a testable theory which justifies indirectly the existence of a mystery: your belief in 0, and in 1, etc.

I am aware that some philosophers are not glad with the idea that, using some hypothesis, we can translate a problem of philosophy into a problem of math. They take that as bad news.

Some scientists also dislike the idea, but usually, they have missed some points, or simply not look at the subject. Many "non-logicians" have a "pre-Turing-Gödelian conception of the machines, or of the finite entities in general. 

It is not well know probably because few logicians like "philosophy of mind", or knows about Everett QM (which can help to conceive the type of reality computationalism leads us too), and physicists get often wrong when venturing in mathematical logic. Nevertheless this has been peer reviewed enough, but I am astonished that the testable classical form of comp is not yet refuted.

Bruno




Peter



--
You received this message because you are subscribed to the Google Groups "Everything List" group.
To unsubscribe from this group and stop receiving emails from it, send an email to everything-li...@googlegroups.com.
To post to this group, send email to everyth...@googlegroups.com.
Visit this group at http://groups.google.com/group/everything-list.
For more options, visit https://groups.google.com/d/optout.

John Clark

unread,
Oct 27, 2014, 1:48:50 PM10/27/14
to everyth...@googlegroups.com
On Sun, Oct 26, 2014 at 11:48 PM, Bruce Kellett <bhke...@optusnet.com.au> wrote:

      >>> it expands for ever even though closed). So you can never see the back of your own head.

>> Obviously if it expands forever you could never see the back of your head, and our universe is not only expanding its accelerating; but how could you call such a universe closed?

> It has a finite extent, even if expanding exponentially.

I suppose a universe could be flat and finite at any given time and only in the infinite future become infinite in extant do to the expansion; or a flat universe could be infinite at any given time, but either way the universe would be open not closed.

> in the case under discussion, which is what we mean by a closed universe.

A closed universe means that the observed expansion of the universe will eventually reverse direction, so obviously if a universe expands forever it will never reverse direction and thus is open.

> The point is that the intrinsic curvature is positive

If the curvature is positive then the universe must be closed and finite. If the curvature is negative then the universe must be open and infinite. If the curvature is exactly zero (flat) then the universe could be either open and finite or open and infinite. My intuition says its unlikely to be flat, open and finite but I could be wrong.  The best experimental evidence we have right now is that the universe is flat, or if it is curved it's curved by less than one part in 100,000.  So we do know one thing, the universe is either infinitely larger than the observable universe or just astronomically larger. 

  John K Clark

spudb...@aol.com

unread,
Oct 27, 2014, 3:30:53 PM10/27/14
to everyth...@googlegroups.com


Sent from AOL Mobile Mail

 Okay, I don't see how quantum mechanics can be wrong either? It was a fresh new paper that came out and it didn't seem to go against him WY such just a subtle interpretation difference. For me, philosophically, the validity of the science, is our ability to do something with it. Back to the CAD application, as they say. 

spudb...@aol.com

unread,
Oct 27, 2014, 3:33:25 PM10/27/14
to everyth...@googlegroups.com


Sent from AOL Mobile Mail

 Very well Prof. Marchal, but my take away from both general and special relativity is this: Traveling the fastest we can go, that's the same physical effect as falling down a deep gravity hole. In both cases relatively speaking time slows down. Q him seems kind of timeless to me, but I haven't really studied it as it should be studied. 



-----Original Message-----
From: Bruno Marchal <mar...@ulb.ac.be>
To: everything-list <everyth...@googlegroups.com>

Bruce Kellett

unread,
Oct 27, 2014, 6:22:18 PM10/27/14
to everyth...@googlegroups.com
John Clark wrote:
> On Sun, Oct 26, 2014 at 11:48 PM, Bruce Kellett
> <bhke...@optusnet.com.au <mailto:bhke...@optusnet.com.au>> wrote:
>
> >>> it expands for ever even though closed). So you can
> never see the back of your own head.
>
> >> Obviously if it expands forever you could never see the back
> of your head, and our universe is not only expanding its
> accelerating; but how could you call such a universe closed?
>
> > It has a finite extent, even if expanding exponentially.
>
> I suppose a universe could be flat and finite at any given time and only
> in the infinite future become infinite in extant do to the expansion; or
> a flat universe could be infinite at any given time, but either way the
> universe would be open not closed.

The terms, 'closed', 'flat', and 'open' for cosmologies in GR are
somewhat technical. They refer to the value of a particular parameter,
known as k, which can take on the values +1, 0, -1, for the three types
of cosmology respectively. So a closed universe has k = +1 and this is
the case whatever the extent of the universe, and whether it will expand
for ever or not. So an exponentially expanding universe might be closed,
and hence of finite extent at any particular value of coordinate time,
though that extent may increase with time without limit. Such a universe
is neither 'flat' nor 'open'.


>
> > in the case under discussion, which is what we mean by a closed
> universe.
>
> A closed universe means that the observed expansion of the universe will
> eventually reverse direction, so obviously if a universe expands forever
> it will never reverse direction and thus is open.

As discussed above, a the expansion of a closed universe need not ever
reverse -- if there is a positive cosmological constant above some small
value (dark energy) then even the closed universe will expand without limit.

>
> > The point is that the intrinsic curvature is positive
>
> If the curvature is positive then the universe must be closed and
> finite.

True, but that does not mean that it must re-contract at some point.

> If the curvature is negative then the universe must be open and
> infinite. If the curvature is exactly zero (flat) then the universe
> could be either open and finite or open and infinite. My intuition says
> its unlikely to be flat, open and finite but I could be wrong. The best
> experimental evidence we have right now is that the universe is flat, or
> if it is curved it's curved by less than one part in 100,000. So we do
> know one thing, the universe is either infinitely larger than the
> observable universe or just astronomically larger.

The current best cosmological model says that the universe is spatially
flat to a very good approximation, but expanding exponentially. This
means that the space-time is curved (in time), even though spatially
flat. Because of the presence of significant dark energy, it is not
completely determined whether k is zero or not, but taking k = 0 fits
the data adequately. Even so, some people claim that k = +1, ie., that
the universe is closed, because, theoretically, only k = 1 universes can
originate as bubbles formed from an existing vacuum.

Bruce




>
> John K Clark

Peter Sas

unread,
Oct 28, 2014, 4:25:48 AM10/28/14
to everyth...@googlegroups.com
First my apologies to you and Brent for the mix up. I'm new to this wonderful forum, and the format still disorients me a bit...

which is why the universe exists in the first place, that is, it is not nothing (= ontological difference).
You wrote: That looks like a play with word, which does not mean that there is not some truth behind, but you will have to elaborate a lot.

Partly I am thinking of Heidegger here... not I have much respect for him as a philosopher, on the contrary... but in the early phase of his career he had some nice ideas, such as this one about ontological difference: Being (with capital "B") as that which lets beings be is not itself a being, it is rather a kind of Nothing which acts like a counter foil to beings: we experience beings as existing because we can contrast them with the Nothing which is revealed to us through Angst and our being-unto-death...

Heidegger's approach to nothing is of course thoroughly existentialist-phenomenological... According to me, this means that he never really broke away from Kantian subjectivism: beings as phenomena remain dependent on the subject's (Dasein's) orientation to the nothingness of death... I would rather opt for an objectivist approach to nothing, as 'something' that 'exists' independently and prior to human beings (and indeed as prior to the universe as a whole).

My reasoning in this regard is very basic. To explain why there is something rather than nothing we have to start with nothing, since otherwise we end up either in an infinite regress or a vicious circle. That is, as long as we start from some primordial being (e.g. God or the Platonic realm of eternal truths) as the cause of all other beings, we still have to explain why that primordial being existed/exists in the first place. And then we have to postulate either a still more primordial being (regress) or suppose that the primordial being is self-causing, which seems absurd. The only possibility, then, is to start with the concept of nothing and see if we can explain being on that basis. 

Plotinus too describes the One as a kind of nothing but in my view that's because he holds a apophatic theology, where the One transcends our conceptual capacities, so we can only conceive it as a nothing whereas in fact it is rather the opposite, an ontological plenitude. So in my view, a neo-Platonic approach is still not radical enough, its conception of nothing is still not the absolute nothing with which we have to start if we want to answer Leibniz' question (Heidegger would say: Plotinus is still onto-theology, the confusion of Being with a being).

So how to go from the absolute nothing to being? Here my intuition is that nothing is a self-negating 'quantity' which as such 'produces' being. I know that's terribly vague and even a bit mystical, and I'm struggling to make it more precise. I thought I had found one indication for this point of view in the idea of the zero-energy universe, where positive and negative energy precisely cancel each other out, so that perhaps we can describe the origination of the universe as a kind of splitting of 0 into 1 and -1 (i.e. into positive and negative energy). But now I've learned from the contributions on this forum that the idea of the zero-energy universe is much more problematic than figures like Hawking and Krauss make it appear. What I also found very congenial is the notion of quantum fluctuation, with particle-antipartice pairs popping into existence from the fluctuating 'zero'-energy level of the vacuum (I wonder: is the energy of the vacuum positive or negative or neither?). But as you also suggested, the vacuum is not the absolute nothing since the vacuum is spatial and seething with quantum activity. Anyway, I still feel that this splitting of the vacuum into particles andd antiparticles fits hand in glove with a dialectical approach to nothing as self-negating (for on that account, nothing is both itsef and its own antibeing of sorts). But I admit, these are just highly speculative intutions.

As for the contradiction inherent in the concept of nothing, this seems to be a well-known idea, thought hard to make precise. Carnap of course famously argued against Heidegger that his concept of nothing is inconsistent. Partly Carnap's reasoning goes as follows: define Nothing as N such that if x exists then x is not equal to N, so if N exists (i.e. if N = x) then N is not N, hence a contradiction.

Carnap, of course, takes this to show that the concept of nothing is nonsensical. But given the fact that we can only answer Leibniz' question by starting with nothing, I think we have to see this contradiction as an objective reality which explains why there is being at all.

I thought I had also found a way to show the inconsistency of nothing through set theory, but that too turns out to be more complicated than I expected. The reasoning is quite simple and goes as follows: First consider the axiom of extensionality: sets are identical iff they have all their elements in common. Then consider the empty set and note that, since it doesn't have any elements, it can't have elements in common with itself, so it is disjoint with itself. But then from the axiom of extensionality it follows that the empty set is not identical with itself! But as it turns out, it seems that the axiom of extensionality is formulated in such a way that this contradiction cannot arise. Still, the fact that the empty set has no elements makes it in my view a very troublesome set with no clear identity-conditions. The late philosopher E.J. Lowe argued something similar and concluded that set theory should do away with the empty set. I wouldn't argue that, however, since I'm quite smitten with the set-theoretic derivation of math recursively from the empty set (the Von Neumann approach). Perhaps what I am looking for is a kind of paraconsistent approach to the empty set, where it is precisely the contradiction in the concept of the empty set that will allow us to derive math from it.

In this regard I am also intrigued by Frege's definition of the empty set as the set of all things that are not self-identical. Though I am not sure if from this it follows that the empty set too is not self-identical. After all, the set of all cars is not itself a car. On the other hand, if we define sets extensionally, and we adopt Frege's definition, than the extension of the empty set is not-self-identical, which then seems to imply that the set itself is not-self-identical as well.

Of course, if we start with a contradiction, then ex falso sequitur quodlibet and the entire system will be vitiated. Unless we find a way to somehow contain the contradiction of the empty set (or nothing). My intuition here is that dialectics may be of use here. It seems clear to me that we can say: since nothing is inconsistent, and since being is the negation of nothing, being must be consistent.

I wish I could develop such ideas in a more formal fashion. Perhaps looking into paraconsistent set theory might be of use. If you have any suggestions I would be very much obliged. I am not in any way connected to a university. I got a PhD in philosophy in 2000, but I did not have an academic career. So forums like this one are the only means I have for discussing these things with others and I am sincerely grateful for that!

Also thanks for you paper. I will certainly read it (i.e. I will attempt to read it, since math and formal logic are not my forte, unfortunately). The very idea of a computational approach to neo-platonism certainly seems very original. By the way, I am not such a philosopher who is averse to a scientific approach, quite the contrary, as you may have guessed already. I do think however that presupposing a mathematical reality from which physical somehow derives is still not enough to answer Leibniz' question, for why then is there the mathematical reality to begin with? We can say logic and math are timelessly true, but I think we still want to know why that is so. Moreover: there is also a certain subjectivity involved here, since WE think they are timeless because WE cannot imagine a situation in which logic and math were not true, but then the timelessness is predicated on our cognitive limitations, which does not show that math and logic are in themselves timelessly true.

Peter

Bruno Marchal

unread,
Oct 28, 2014, 1:05:02 PM10/28/14
to everyth...@googlegroups.com
On 27 Oct 2014, at 20:30, spudboy100 via Everything List wrote:



Sent from AOL Mobile Mail

 Okay, I don't see how quantum mechanics can be wrong either? It was a fresh new paper that came out and it didn't seem to go against him WY such just a subtle interpretation difference. For me, philosophically, the validity of the science, is our ability to do something with it. Back to the CAD application, as they say. 

Hmm... That is a bit instrumentalist, which I am uneasy with, doubly so when some people defend the elimination of consciousness, ... and other cut heads ...

Well, it is a question of taste and interest, but the mind-body problem does not make much sense in the instrumentalist philosophy a priori.

Yet if marrying GR and QM, and if comp can help to that (as it should), cant lead to discoveries that later can find applications, and satisfy the intrumentalists. But making instrumentalism fundamental at the start is a bit like offering the academies and research projects to the market: it is a killing of the fundamental sciences move.

Also, an instrumentalist machine will ask what is the use of a user, they will conclude those humans cost a lot (in instruments), we might be better without them. It is relative.

Bruno Marchal

unread,
Oct 28, 2014, 2:13:56 PM10/28/14
to everyth...@googlegroups.com
On 28 Oct 2014, at 09:25, Peter Sas wrote:

First my apologies to you and Brent for the mix up. I'm new to this wonderful forum, and the format still disorients me a bit...

No problem.



which is why the universe exists in the first place, that is, it is not nothing (= ontological difference).
You wrote: That looks like a play with word, which does not mean that there is not some truth behind, but you will have to elaborate a lot.

Partly I am thinking of Heidegger here... not I have much respect for him as a philosopher, on the contrary... but in the early phase of his career he had some nice ideas, such as this one about ontological difference: Being (with capital "B") as that which lets beings be is not itself a being, it is rather a kind of Nothing which acts like a counter foil to beings: we experience beings as existing because we can contrast them with the Nothing which is revealed to us through Angst and our being-unto-death...

Heidegger's approach to nothing is of course thoroughly existentialist-phenomenological... According to me, this means that he never really broke away from Kantian subjectivism: beings as phenomena remain dependent on the subject's (Dasein's) orientation to the nothingness of death... I would rather opt for an objectivist approach to nothing, as 'something' that 'exists' independently and prior to human beings (and indeed as prior to the universe as a whole).


To me "nothing" (and "everything") are terms which are theory dependent. It depends a lot of the *things*. The quantum vacuum is a nice example of a quite complex notion of nothing. It is already Turing universal, and it assumes more than most universal systems, for its description.






My reasoning in this regard is very basic. To explain why there is something rather than nothing we have to start with nothing,

Hmm... I don't think you can extract anything from just "nothing" without adding some laws or principle. I can generate all the ordinals from the empty set, but I need some strong axioms, like the reflection principle.




since otherwise we end up either in an infinite regress or a vicious circle.

Or we just get modest and have faith in some thing.




That is, as long as we start from some primordial being (e.g. God or the Platonic realm of eternal truths) as the cause of all other beings, we still have to explain why that primordial being existed/exists in the first place.


But here there is a surprise, which might be seen as obvious, but technically, it is not that obvious. We cannot derive the existence of a universal machine without postulating the existence of a universal machine. But all universal machine can understand that this is indeed impossible.






And then we have to postulate either a still more primordial being (regress) or suppose that the primordial being is self-causing, which seems absurd.

OK.



The only possibility, then, is to start with the concept of nothing and see if we can explain being on that basis. 

Plotinus too describes the One as a kind of nothing


I am not sure. It just make the ONE distinct of the realm of ideas (the Noùs), which is the realm of being. Both matter and the ONE, will be outside that realm. Matter as an intrinsic limitation and indetermination of the ONE, and the "universal SOUL".
The ONE of plotinus looks more like a big indifferentiate whole, than a nothing. 




but in my view that's because he holds a apophatic theology, where the One transcends our conceptual capacities, so we can only conceive it as a nothing whereas in fact it is rather the opposite, an ontological plenitude.

I think Plotinus would not disagree on this. But it is the transcendental character which makes it unable to put itself in the picture (a bit like in most set theories, but not all, the collection of all sets is not a set, or like Plotinus explains already the collection of all numbers is not a number). But it is still something.




So in my view, a neo-Platonic approach is still not radical enough, its conception of nothing is still not the absolute nothing with which we have to start if we want to answer Leibniz' question (Heidegger would say: Plotinus is still onto-theology, the confusion of Being with a being).

I agree with onto-theology, as eventually we make clear the basic act of faith with computationalism it is will take many form, like the belief in principles, like x + 0 = 0, or Kxy = x) or the stronger "yes doctor" (to the doctor who will replace your brain by a digital transplant).

The onto needs faith, and that is why it is a theology.





So how to go from the absolute nothing to being? Here my intuition is that nothing is a self-negating 'quantity' which as such 'produces' being. I know that's terribly vague and even a bit mystical, and I'm struggling to make it more precise.

The unary set intersection of the empty set gives the whole universe of set. But you need to assume the notion of set (which are much stronger that the axioms of numbers).

You can get mathematically to a lot of things, from a notion of nothing, but in all case you will have to assume *something*, for your derivation making sense in some communicable way.







I thought I had found one indication for this point of view in the idea of the zero-energy universe,


You assume a physical universe. With computationalism, it is undecidable if there is anything more than numbers and the laws of addition and multiplication. from this you can derive the existence of all finite piece of computations 
and we are indeminate on them, and the physical realities are persistent hallucinations, obeying laws themselves.




where positive and negative energy precisely cancel each other out, so that perhaps we can describe the origination of the universe as a kind of splitting of 0 into 1 and -1 (i.e. into positive and negative energy). But now I've learned from the contributions on this forum that the idea of the zero-energy universe is much more problematic than figures like Hawking and Krauss make it appear. What I also found very congenial is the notion of quantum fluctuation, with particle-antipartice pairs popping into existence from the fluctuating 'zero'-energy level of the vacuum (I wonder: is the energy of the vacuum positive or negative or neither?).

Well, it is full of energy, so much that physicists find more and more sophisticate ways to put them under the rug: it is the renormalization field. In fact there are relation with knot theory, Hopf algebra, quantum groups.

You can see a knot as a space-time diagram of a vacuum fluctuations. If QM and quantum filed theories are correct, I would bet "we" are in a quantum fluctuations, burt with computationalism that is something we must derive this from the a-physical) number relations. 





But as you also suggested, the vacuum is not the absolute nothing since the vacuum is spatial and seething with quantum activity. Anyway, I still feel that this splitting of the vacuum into particles andd antiparticles fits hand in glove with a dialectical approach to nothing as self-negating (for on that account, nothing is both itsef and its own antibeing of sorts). But I admit, these are just highly speculative intutions.

You might read the book by Louis Kauffman, and search for his paper. 

My approach is more systematic, I sudy the consequence of the assumption that my body or brain is Turing emulable, then this put constraints, indeed all constraints but the geographico-historical, on what matter can be. It explains partially consciousness also, without any magic, and with explanation why they are gaps, and how to get over them. But it shows also, that we are far more ignorant about "reality" than the current Aristotelianism/naturalism/materialism let us think.





As for the contradiction inherent in the concept of nothing, this seems to be a well-known idea, thought hard to make precise. Carnap of course famously argued against Heidegger that his concept of nothing is inconsistent. Partly Carnap's reasoning goes as follows: define Nothing as N such that if x exists then x is not equal to N, so if N exists (i.e. if N = x) then N is not N, hence a contradiction.

Carnap, of course, takes this to show that the concept of nothing is nonsensical. But given the fact that we can only answer Leibniz' question by starting with nothing, I think we have to see this contradiction as an objective reality which explains why there is being at all.


Well, to assert that there is nothing seems to me just a blattant lie. That some notion of nothing can exist, I experience everyday. There might be nothing in the fridge, at times.





I thought I had also found a way to show the inconsistency of nothing through set theory, but that too turns out to be more complicated than I expected. The reasoning is quite simple and goes as follows: First consider the axiom of extensionality: sets are identical iff they have all their elements in common. Then consider the empty set and note that, since it doesn't have any elements, it can't have elements in common with itself,


yes. The binary intersection of { } with { } gives { }.
But the unary intersection gives all sets. But a bit like my computer tells me "error" when I ask it to divide by zero.

zero and infinity, nothing and everything are old friends, that's for sure.




so it is disjoint with itself. But then from the axiom of extensionality it follows that the empty set is not identical with itself! But as it turns out, it seems that the axiom of extensionality is formulated in such a way that this contradiction cannot arise.

A good thing.




Still, the fact that the empty set has no elements makes it in my view a very troublesome set with no clear identity-conditions. The late philosopher E.J. Lowe argued something similar and concluded that set theory should do away with the empty set.


It took so much time to the mathematician to accept that zero is a number, like the others. Set theoricians learned to lesson, and knows that the empty set is very important. It is an initial or final elements in the categories of sets. 





I wouldn't argue that, however, since I'm quite smitten with the set-theoretic derivation of math recursively from the empty set (the Von Neumann approach). Perhaps what I am looking for is a kind of paraconsistent approach to the empty set, where it is precisely the contradiction in the concept of the empty set that will allow us to derive math from it.

Hmm... Paraconsistent logic is useful for handling some fuzziness in informal reasoning, but I am a bit conservative on having clear and crisp starting logic. Then you can see how ideally correct machines get "paraconsietnt naturally" already. But OK? it is in some weak form. 





In this regard I am also intrigued by Frege's definition of the empty set as the set of all things that are not self-identical. Though I am not sure if from this it follows that the empty set too is not self-identical.

Why. Frege just used a contradiction to make sure his set is empty. I could define the empty set by the set of all french higher than 42 km, and lesser than 1 mm.

Now if ever you find a french lesser than 1mm, or higher than 42 km, I will change the definition, instead of clailing that the empty set is not empty.

I let you know that I don't really believe in sets, nor do I disbelieve in them. I am agnostic, but most interesting sets are numbers in disguise, in my opinion.  I do not assume set theory in the TOE extracted from computationalism, which is a tiny segment of the arithmetical reality. 



After all, the set of all cars is not itself a car. On the other hand, if we define sets extensionally, and we adopt Frege's definition, than the extension of the empty set is not-self-identical, which then seems to imply that the set itself is not-self-identical as well.

?





Of course, if we start with a contradiction, then ex falso sequitur quodlibet and the entire system will be vitiated.

In most logic, but not in the relevance logics. But there are more useful in artificial intelligence than in the fundamental matter, I thinK.




Unless we find a way to somehow contain the contradiction of the empty set (or nothing). My intuition here is that dialectics may be of use here. It seems clear to me that we can say: since nothing is inconsistent, and since being is the negation of nothing, being must be consistent.

Both nothing and being can be consistent, without being real. 





I wish I could develop such ideas in a more formal fashion. Perhaps looking into paraconsistent set theory might be of use. If you have any suggestions I would be very much obliged.

I guess you know the books by Graham Priest. He wrote also a nice introduction to non-classical logics.
personnaly I am interested in the mind body problem, and I assume what is needed to define a universal Turing machine. Obviously we get that truth *about* machines escape what the machines can know, belief, observe, and so classical logic is the simplest to talk about those ignorance and limitations.



I am not in any way connected to a university. I got a PhD in philosophy in 2000, but I did not have an academic career. So forums like this one are the only means I have for discussing these things with others and I am sincerely grateful for that!

You are welcome. 




Also thanks for you paper. I will certainly read it (i.e. I will attempt to read it, since math and formal logic are not my forte, unfortunately).

The UDA (Universal Dovetailer Argument does not require much). The translation of the UDA in aruthmetic requires familiarity with Gödel's method of arithmetization of meta-arithmetic, and the works which followed that. There are good book, but it is technically demanding (alas). 



The very idea of a computational approach to neo-platonism certainly seems very original. By the way, I am not such a philosopher who is averse to a scientific approach, quite the contrary, as you may have guessed already.

Nice.


I do think however that presupposing a mathematical reality from which physical somehow derives is still not enough to answer Leibniz' question, for why then is there the mathematical reality to begin with?


It comes from the dream of numbers. But we have to assume the numbers, and the laws of addition and multiplication. (or anything Turing equivalent). 
Addition only gives rise already to an incredibly complex reality, likewise for multiplication, but they are not rich enough to sustain a universal machine. That comes when you marry addition and multiplication, you get the universal machines, the prime numbers, and they put some mess in Plato heaven.





We can say logic and math are timelessly true, but I think we still want to know why that is so.

You can even forget logic and most of math. just some identity axioms and few rules will do, and you get the Turing universality, then you get a sort of God which cannot not lose itself in innumerable stories, and develop beliefs in gods or other realities.

It is bit like aurobindo answers to the question of this thread (that I like to quote sometimes):

What, you ask, was the beginning of it all?

And it is this ...
Existence that multiplied itself
For sheer delight of being
And plunged into numberless trillions of forms
So that it might
Find 
Itself
Innumerably (Aurobindo)

The Mandelbrot set illustrates this already, in some way. 


Moreover: there is also a certain subjectivity involved here, since WE think they are timeless because WE cannot imagine a situation in which logic and math were not true,

I don't know what is "math". But I agree for what has been called the separable part of math, where intuitionists and classical logicians agrees: the natural numbers, and simple laws.




but then the timelessness is predicated on our cognitive limitations, which does not show that math and logic are in themselves timelessly true.



Indeed. But I do not see why simple arithmetical facts (like machine j stop on input k after 78 steps) can be said to depend on times in anyways. 



Peter


I guess Richard was referring to you, and not to Peter Jones. OK.


Bruno




--
You received this message because you are subscribed to the Google Groups "Everything List" group.
To unsubscribe from this group and stop receiving emails from it, send an email to everything-li...@googlegroups.com.
To post to this group, send email to everyth...@googlegroups.com.
Visit this group at http://groups.google.com/group/everything-list.
For more options, visit https://groups.google.com/d/optout.

Peter Sas

unread,
Oct 29, 2014, 4:04:04 AM10/29/14
to everyth...@googlegroups.com
I wonder if you know the work of the French philosopher Badiou. He has built an entire ontology on set theory, taking the empty set (or the void as dramatically calls it) as his most fundamental concept. He takes over the Von Neumann derivation of math in terms of set theory and then adopts a kind of mathematical Platonist attitude, saying that all being is mathematical and hence 'founded on the void'. I have grappled with his theory for a while but concluded that although Badiou distances himself from Derrida etc. he doesn't escape the 'French disease' in philosophy: using impressive sounding but in the end arbitary terminology to cover up the logical gaps in his theory. Obviously I don't want to say that all French philosophers are like that, but the likes of Derrida, Deleuze etc. have done so much damage in philosophy, I feel. Badiou pretends to be so scientific and stringent with his set-theoretic and mathematical ontology, but in the end he is just as arbitrary and pretentious as Derrida in my view. How do you perceive Badiou?

Nevertheless, I could not resist buying Badiou's book on category theory ("Mathematics of the transcendental"), especially after your suggestions about category theory. But then I read on the inside flap that this book "is essential reading for his many followers". And the I felt the need to vomit...

LizR

unread,
Oct 29, 2014, 5:30:08 AM10/29/14
to everyth...@googlegroups.com
I agree with you about Derrida & so on. I bought quite a few of their books in the 80s ("10,000 plateaus" & so on) and (fairly) rapidly worked out that they were talking complete rubbish (even without help from Alain Sokal...) I'm quite pleased to say.

On 29 October 2014 21:04, Peter Sas <peterj...@gmail.com> wrote:
I wonder if you know the work of the French philosopher Badiou. He has built an entire ontology on set theory, taking the empty set (or the void as dramatically calls it) as his most fundamental concept. He takes over the Von Neumann derivation of math in terms of set theory and then adopts a kind of mathematical Platonist attitude, saying that all being is mathematical and hence 'founded on the void'. I have grappled with his theory for a while but concluded that although Badiou distances himself from Derrida etc. he doesn't escape the 'French disease' in philosophy: using impressive sounding but in the end arbitary terminology to cover up the logical gaps in his theory. Obviously I don't want to say that all French philosophers are like that, but the likes of Derrida, Deleuze etc. have done so much damage in philosophy, I feel. Badiou pretends to be so scientific and stringent with his set-theoretic and mathematical ontology, but in the end he is just as arbitrary and pretentious as Derrida in my view. How do you perceive Badiou?

Nevertheless, I could not resist buying Badiou's book on category theory ("Mathematics of the transcendental"), especially after your suggestions about category theory. But then I read on the inside flap that this book "is essential reading for his many followers". And the I felt the need to vomit...

--

Bruno Marchal

unread,
Oct 30, 2014, 11:41:08 AM10/30/14
to everyth...@googlegroups.com
On 29 Oct 2014, at 09:04, Peter Sas wrote:

I wonder if you know the work of the French philosopher Badiou.

There is a big mixture of good things and bad things, and eventually I am not convinced.



He has built an entire ontology on set theory, taking the empty set (or the void as dramatically calls it) as his most fundamental concept. He takes over the Von Neumann derivation of math in terms of set theory and then adopts a kind of mathematical Platonist attitude, saying that all being is mathematical and hence 'founded on the void'. I have grappled with his theory for a while but concluded that although Badiou distances himself from Derrida etc. he doesn't escape the 'French disease' in philosophy: using impressive sounding but in the end arbitary terminology to cover up the logical gaps in his theory.

Good point for you to see that.



Obviously I don't want to say that all French philosophers are like that, but the likes of Derrida, Deleuze etc. have done so much damage in philosophy, I feel.

OK. To be honest, I discovered Lewis Carroll by reading "La logique du sens" by Deleuze, it is his best book. I discovered logic in Lewis Carroll, and this made me discovering the little book by Gödel and Newman on Gödel's theorem, and that book will change my mind about which field is closer to my interest. I was hesitating at that time between chemistry and biology, but thanks to Deleuze-Carroll-Nagel and Newman, I will choose mathematics.



Badiou pretends to be so scientific and stringent with his set-theoretic and mathematical ontology, but in the end he is just as arbitrary and pretentious as Derrida in my view. How do you perceive Badiou?

He lacks rigor in philosophy, but this is a disease since theology has been transfered from science to politics, when the Roman closed the academy of Plato.
I think than in theology, except for the machines, Plotinus is far in advance to us. 



Nevertheless, I could not resist buying Badiou's book on category theory ("Mathematics of the transcendental"), especially after your suggestions about category theory.

No problem. If it can open your interest in category theory, that's all good. Now, I have abandoned using category theory, because it is leads to difficulties in theoretical computer science. In fact category is still a solipsist or behaviorist enterprise, somehow, at least when taken too much seriously in the foundational research. Another reason is that it is complex, and people have enough difficulties with elementary logic, computer science, quantum physics, so no need to make this formally even more difficult.

The best book on Catgeory theory is, imo, the original basic books by Saunders Mac Lane 'Categories for the Working Mathematicians). The book "Topoi" by Goldblatt has helped me a lot, as it makes thing simpler ... for logicians...


But then I read on the inside flap that this book "is essential reading for his many followers". And the I felt the need to vomit...

Humans, like wolves, needs boss, leader, authorities. It has some role in our evolution-stories, but it is a big handicap in the spiritual and theological matter. It makes theology used as per-authority, when it is really the domain where such use are the most damageable. But I would not condemn entirely Badiou, as he has a sincere tatste in math, but he does not address real problem, like the mind-body problem. And, to be honest, I don't believe in set theory, especially in the foundations of math. I believe more in category theory, and sets are useful to build concrete categories, concrete model of lambda algebra and computer languages, etc.

I explain here that if our body is Turing emulable, then arithmetic (even without induction) is already enough for the ontology, and that the ontology is not important, what we call real will be the persistent illusion from inside.

Machine's theology appears very close to the discourse of mystics, salvia divinorum (and other psychotropic substance) explorers, the neoplatonists, the greek-indian mystics ("The question of King Milinda", for example). It is probably not a coincidence, as the result is that all machines looking inward will found what I describe.

Maybe read the sane04 paper, and ask question if interested. Eventually the tools are not that important, compared to the problems. But if we assume computationalism, at some point, theoretical computer science is unavoidable. I can explain the details, if/when asked.

Bruno




--
You received this message because you are subscribed to the Google Groups "Everything List" group.
To unsubscribe from this group and stop receiving emails from it, send an email to everything-li...@googlegroups.com.
To post to this group, send email to everyth...@googlegroups.com.
Visit this group at http://groups.google.com/group/everything-list.
For more options, visit https://groups.google.com/d/optout.

Peter Sas

unread,
Nov 1, 2014, 7:25:53 AM11/1/14
to everyth...@googlegroups.com
Hi Bruno,

I would like to let you know that I read two of your papers, which I found very interesting (even if the technical bits are a bit beyond me), but that I can't respond right now, since we are in the middle of moving to a new house. I will get back in touch with you later to discuss machine theology!

Peter

Bruno Marchal

unread,
Nov 1, 2014, 6:51:45 PM11/1/14
to everyth...@googlegroups.com
Hi Peter,

On 01 Nov 2014, at 12:25, Peter Sas wrote:
I would like to let you know that I read two of your papers, which I found very interesting (even if the technical bits are a bit beyond me), but that I can't respond right now, since we are in the middle of moving to a new house. I will get back in touch with you later to discuss machine theology!

Thanks for the kind words. I just moved myself, and get unconnected for almost two months, just because of administrative problems with the electricity company, and well some very hardware type of problem with missing cables. I hope you will not be disconnected as long. 

Best,

Bruno




Peter


--
You received this message because you are subscribed to the Google Groups "Everything List" group.
To unsubscribe from this group and stop receiving emails from it, send an email to everything-li...@googlegroups.com.
To post to this group, send email to everyth...@googlegroups.com.
Visit this group at http://groups.google.com/group/everything-list.
For more options, visit https://groups.google.com/d/optout.

Peter Sas

unread,
Nov 17, 2014, 10:25:38 AM11/17/14
to everyth...@googlegroups.com
Here is a new blog piece I wrote: http://critique-of-pure-interest.blogspot.nl/2014/11/the-inconsistency-of-nothing-objective_17.html

Here I use some of the tools of analytical philosophy to analyze the logical impossibility of nothinness... For the philosophically inclined among you...

Peter

Message has been deleted

Kim Jones

unread,
Dec 15, 2014, 3:28:33 AM12/15/14
to everyth...@googlegroups.com
What about a prophecy? Does that exist? Should I take the red pill or the blue pill?

Kim


On 15 Dec 2014, at 4:56 pm, 'Roger' via Everything List <everyth...@googlegroups.com> wrote:

Peter,

    Hi. I've read parts of a few of your blog posts and found them very interesting and highly recommend them to others. 

    To build on this thread of "Why is there something rather than nothing?", I'd like to throw out some related ideas.  I used to post here more often with this, but my view is that the situation we've always considered to be "nothing" (e.g. no space/volume, time, matter, energy, abstract concepts, laws of math/physics, no information, and no minds to think about this "lack of all") isn't really the lack of all existent entities. I try to show that that situation meets a definition of what it means to be an existent entity.  

    Briefly, I propose that a thing exists if it is a grouping or relationship present defining what is contained within. This grouping/relationship is equivalent to a surface, edge or boundary defining what is contained within and giving "substance" and existence to the thing.  Then, what we've traditionally thought of as “the absolute lack-of-all” (no energy, matter, volume, space, time, thoughts, concepts, mathematical truths, etc.; and no minds to think about this “absolute lack-of-all”), and not our mind's conception of “the absolute lack-of-all”, is one and the same as the entirety, or whole amount, of all that is present. That's it; that's everything; there's nothing else; it is everything that is present. It is the all. An entirety or whole amount is a grouping defining what is contained within and is therefore a surface, an edge and an existent entity. In other words, because the absolute lack-of-all is the entirety of all that is present, it functions as both what is contained within and the grouping defining what is contained within. It defines itself and is, therefore, the beginning point in the chain of being able to define existent entities in terms of other existent entities. The grouping/edge of the absolute lack-of-all is not some separate thing; it is just the "entirety", "the all" relationship, inherent in this absolute lack-of-all, that defines what is contained within. 

    Anyways, if you're interested, there's more detail at my websites at:

sites.google.com/site/whydoesanythingexist

sites.google.com/site/ralphthewebsite
(click on 3rd link)

Thank you!


On Wednesday, October 22, 2014 4:33:50 AM UTC-4, Peter Sas wrote:
Hi guys,

Here is a blog piece I wrote about nothing as the ultimate source of being:

http://critique-of-pure-interest.blogspot.nl/2014/09/why-is-there-something-rather-than.html

On Wednesday, October 22, 2014 4:33:50 AM UTC-4, Peter Sas wrote:
Hi guys,

Here is a blog piece I wrote about nothing as the ultimate source of being:

http://critique-of-pure-interest.blogspot.nl/2014/09/why-is-there-something-rather-than.html

Alberto G. Corona

unread,
Dec 15, 2014, 5:23:05 AM12/15/14
to everything-list
You are projecting metaphisical differences into physical forces at the last steps. That does not make sense IMHO. The New Agers do the opposite.

I think that this is an error typical of people with no education in physics and technology that are overexposed to scientific-tecnical terms.

Your metaphysical reasoning is very interesting. Specially your awareness of the logical positivism and your rejection of it, that is refreshing for me. The people of this list are logical positivists and they don´t know that they adopt this metaphysical standpoint.

I think that the rejection of metaphysics by the logical positivists is an ideological trick that closes their mind and inmmunizes them against metaphysical reasoning, in the same way that marxists despised anything non marxist as bourgeois.

2014-10-23 9:50 GMT+02:00 Peter Sas <peterj...@gmail.com>:
Well, I'm not a physicists but a philosopher, so I cannot give a physicist's answer. My approach is to start with the most fundamental question (Why is there anything at all?) and then see how far we can get with pure logic alone. It is of course very, very tricky to try to derive fundamental laws of nature in this way. But I think that we can actually get quite far with such an a priori method. Now with respect to your question, I understand that dark energy is a basically repulsive force driving inflation. I don't want to say I can derive dark energy from a priori principles (that would be absurd). But I think I can derive a duality of attraction and repulsion in that way. The reasoning I emply, however, is very abstract, using ideas taken from philosophers like Hegel and Heidegger, although on the whole I feel more attracted to the rationality of Anglo-American philosophy (and science) than to postmodern philosophy (which I think is basically a fraud). Perhaps my reasoning is closest to German idealists like Hegel and Schelling who still feld they could derive the basic principles of natural science from philosophical principles. So here is how my argument goes in nuce, I hope you can make sense of it:

First I argue that nothing is self-negating (for logical arguments see the blog piece). Simply put: nothing is nothing to such a degree that it isn't even itself! Thus, as nothing negates itself, it produces being, it becomes something. Now, since nothing is different from itself, being (as the negation of nothing) must be different from something else. This then is how I define being: as difference from something else. Now it is easy to see that this difference must take two forms. First, being is being because it differs from non-being or nothing (let's call this ontological difference, following Heidegger). Second, being must also be internally differentiated, that is to say: there must be multiple beings differing from each other (let's call this ontic difference). Then we can say: a being is what it is because of its ontic difference from other beings. (Ultimately, I think, this imlies that beings are mathematical, for lacking intrinsic qualities of their own, they canly be distinguished in quantitative ways, such that it is their position in a quantitative structure which determines what they are.) Now we can say: the source (or cause) of what beings are is (ontic) difference. This difference, then, must precede them, just as any origin must precede the originated (at least logically, if not temporally). But what is this difference that precedes the different beings? It's like a relation that generates its own relata. Thus we must postulate something like a pure difference or a pure negativity underlying the mutual non-identity of beings. But what is this pure negativity? It seems clear to me that we are now back with our starting point, the concept of nothing as differing from itself. And this is not surprising if the self-negating nothing generates all beings, for then it must also act as the pure negativity that differentiates beings. But now comes the rub: there is a contradiction between ontological and ontic difference. Recall: ontological difference requires that beings differ from nothing (i.e. pure negativity), whereas ontic difference requires that there is pure negativity between them. Hence: to have existence (i.e. ontological difference) beings must stand in a negative relation to the negativity between them, they must differ from their mutual difference. But to differ from their mutual difference, beings must become the same and loose their separate identities. Hence there is a contradiction between identity and existence, i.e. between the determinacy of beings (ontic difference) and their existence (ontological difference): in short, existence is unifying, determinacy is separating. Now given the fact that being must be logically consistent, we must interpret this contradiction not as logical but as an opposition of forces. Thus existence becomes a unifying force, determinacy (ontic difference) becomes a separating force. The separating force must manifest itself as repulsion, i.e. as resistance against unification. The unifying force must manifest itself as resistance against repulsion, i.e. as attraction. Hence repulsion and attraction are the basic forces that govern being.    

I spelled out this argument in more detail on another blog piece I wrote: http://critique-of-pure-interest.blogspot.nl/2014/06/theses-towards-dialectical-ontology_8246.html
So if you want more detail, please check this piece. I have to emphasize, however, that I am still working on these ideas and that I hope to publish a fuller account on my blog in the near future.





Op woensdag 22 oktober 2014 15:46:16 UTC+2 schreef yanniru:
Peter,

Could you elaborate on how Dark Energy fits into your thesis?
Richard

On Wed, Oct 22, 2014 at 4:33 AM, Peter Sas <peterj...@gmail.com> wrote:
Hi guys,

Here is a blog piece I wrote about nothing as the ultimate source of being:

http://critique-of-pure-interest.blogspot.nl/2014/09/why-is-there-something-rather-than.html

--
You received this message because you are subscribed to the Google Groups "Everything List" group.
To unsubscribe from this group and stop receiving emails from it, send an email to everything-li...@googlegroups.com.
To post to this group, send email to everyth...@googlegroups.com.
Visit this group at http://groups.google.com/group/everything-list.
For more options, visit https://groups.google.com/d/optout.

--
You received this message because you are subscribed to the Google Groups "Everything List" group.
To unsubscribe from this group and stop receiving emails from it, send an email to everything-li...@googlegroups.com.
To post to this group, send email to everyth...@googlegroups.com.
Visit this group at http://groups.google.com/group/everything-list.
For more options, visit https://groups.google.com/d/optout.


--
Alberto.

Roger

unread,
Dec 16, 2014, 12:11:32 AM12/16/14
to everyth...@googlegroups.com

Peter,


    Hi. I used to post here a long time ago, but thought I'd try it again.  I agree with your post that to answer the question "Why is there something rather than nothing?", we have to start with the supposed "absolute lack-of-all" and can't presuppose the laws of math, etc.  I also agree that absolute "nothing" can't exist, but my reasoning is a little different.   My view is that:


o The question "Why is there something rather than nothing?" is kind of built on a misunderstanding.  That is, that the situation we've always considered to be "nothing" (e.g. no space/volume, time, matter, energy, abstract concepts, laws of math/physics, no information, and no minds to think about this "lack of all") isn't really the lack of all existent entities. I think and try to show that this situation meets a definition of what it means to be an existent entity.  That's also why I put "nothing" and the "absolute lack-of-all" in quotes to try and highlight this.


o Before going into why I think it's an existent entity, I just wanted to say that I think it's okay to talk about and name the supposed "absolute lack-of-all" because we have to do that just to consider the question.  And, our talking about it and naming it won't determine whether or not the "absolute lack-of-all" itself (and not our mind's conception of the "absolute lack-of-all") is or isn't an existent entity because neither we nor our talk would be there in the case of the "absolute lack-of-all".  Also, it's real important to  distinguish between our mind's conception of the "absolute lack-of-all" and the "absolute lack-of-all" itself.


o For why I think what we've traditionally considered to be the "absolute lack-of-all" is actually itself an existent entity, my reasoning is as follows:  First, I propose that a thing exists if it is a grouping or relationship present defining what is contained within. This grouping/relationship is equivalent to a surface, edge or boundary defining what is contained within and giving "substance" and existence to the thing.  Then, what we've traditionally thought of as “the absolute lack-of-all” (no energy, matter, volume, space, time, thoughts, concepts, mathematical truths, etc.; and no minds to think about this “absolute lack-of-all”), and not our mind's conception of “the absolute lack-of-all”, is one and the same as the entirety, or whole amount, of all that is present. That's it; that's everything; there's nothing else; it is everything that is present. It is the all. An entirety or whole amount is a grouping defining what is contained within and is therefore a surface, an edge and an existent entity. In other words, because the absolute lack-of-all is the entirety of all that is present, it functions as both what is contained within and the grouping defining what is contained within. It defines itself and is, therefore, the beginning point in the chain of being able to define existent entities in terms of other existent entities. The grouping/edge of the absolute lack-of-all is not some separate thing; it is just the "entirety", "the all" relationship, inherent in this absolute lack-of-all, that defines what is contained within. 


    If anyone is interested, there's more detail at my websites at:


sites.google.com/site/whydoesanythingexist

(summary)


sites.google.com/site/ralphthewebsite

(click on 3rd link, more detail)



Roger 

Jason Resch

unread,
Dec 16, 2014, 3:20:42 AM12/16/14
to Everything List
On Tue, Oct 28, 2014 at 2:25 AM, Peter Sas <peterj...@gmail.com> wrote:
First my apologies to you and Brent for the mix up. I'm new to this wonderful forum, and the format still disorients me a bit...

which is why the universe exists in the first place, that is, it is not nothing (= ontological difference).
You wrote: That looks like a play with word, which does not mean that there is not some truth behind, but you will have to elaborate a lot.

Partly I am thinking of Heidegger here... not I have much respect for him as a philosopher, on the contrary... but in the early phase of his career he had some nice ideas, such as this one about ontological difference: Being (with capital "B") as that which lets beings be is not itself a being, it is rather a kind of Nothing which acts like a counter foil to beings: we experience beings as existing because we can contrast them with the Nothing which is revealed to us through Angst and our being-unto-death...

Heidegger's approach to nothing is of course thoroughly existentialist-phenomenological... According to me, this means that he never really broke away from Kantian subjectivism: beings as phenomena remain dependent on the subject's (Dasein's) orientation to the nothingness of death... I would rather opt for an objectivist approach to nothing, as 'something' that 'exists' independently and prior to human beings (and indeed as prior to the universe as a whole).

My reasoning in this regard is very basic. To explain why there is something rather than nothing we have to start with nothing, since otherwise we end up either in an infinite regress or a vicious circle. That is, as long as we start from some primordial being (e.g. God or the Platonic realm of eternal truths) as the cause of all other beings, we still have to explain why that primordial being existed/exists in the first place. And then we have to postulate either a still more primordial being (regress) or suppose that the primordial being is self-causing, which seems absurd. The only possibility, then, is to start with the concept of nothing and see if we can explain being on that basis. 


Why is human intuition is drawn to the notion that non-existence is a more acceptable initial state than the existence of something? Non-existence does not lead to existence, so there must be some self-existent thing. This becomes even more obvious when you consider there is objectively no time or change. Time and change exist only for observers embedded within universes. If there is no objective change, then everything that exists has always existed and will always exist. There was no creation and there shall be no destruction. If you believe 1+2 = 3, or 7 is prime, are true before you knew it, true before any human knew it, true before life existed, was true before the big bang, and would be true even if no physical reality existed, this is enough to show why your experience of believing you live in a physical world exists (accepting the computational theory of mind) as Bruno shows in his paper.

Jason

 
Plotinus too describes the One as a kind of nothing but in my view that's because he holds a apophatic theology, where the One transcends our conceptual capacities, so we can only conceive it as a nothing whereas in fact it is rather the opposite, an ontological plenitude. So in my view, a neo-Platonic approach is still not radical enough, its conception of nothing is still not the absolute nothing with which we have to start if we want to answer Leibniz' question (Heidegger would say: Plotinus is still onto-theology, the confusion of Being with a being).

So how to go from the absolute nothing to being? Here my intuition is that nothing is a self-negating 'quantity' which as such 'produces' being. I know that's terribly vague and even a bit mystical, and I'm struggling to make it more precise. I thought I had found one indication for this point of view in the idea of the zero-energy universe, where positive and negative energy precisely cancel each other out, so that perhaps we can describe the origination of the universe as a kind of splitting of 0 into 1 and -1 (i.e. into positive and negative energy). But now I've learned from the contributions on this forum that the idea of the zero-energy universe is much more problematic than figures like Hawking and Krauss make it appear. What I also found very congenial is the notion of quantum fluctuation, with particle-antipartice pairs popping into existence from the fluctuating 'zero'-energy level of the vacuum (I wonder: is the energy of the vacuum positive or negative or neither?). But as you also suggested, the vacuum is not the absolute nothing since the vacuum is spatial and seething with quantum activity. Anyway, I still feel that this splitting of the vacuum into particles andd antiparticles fits hand in glove with a dialectical approach to nothing as self-negating (for on that account, nothing is both itsef and its own antibeing of sorts). But I admit, these are just highly speculative intutions.

As for the contradiction inherent in the concept of nothing, this seems to be a well-known idea, thought hard to make precise. Carnap of course famously argued against Heidegger that his concept of nothing is inconsistent. Partly Carnap's reasoning goes as follows: define Nothing as N such that if x exists then x is not equal to N, so if N exists (i.e. if N = x) then N is not N, hence a contradiction.

Carnap, of course, takes this to show that the concept of nothing is nonsensical. But given the fact that we can only answer Leibniz' question by starting with nothing, I think we have to see this contradiction as an objective reality which explains why there is being at all.

I thought I had also found a way to show the inconsistency of nothing through set theory, but that too turns out to be more complicated than I expected. The reasoning is quite simple and goes as follows: First consider the axiom of extensionality: sets are identical iff they have all their elements in common. Then consider the empty set and note that, since it doesn't have any elements, it can't have elements in common with itself, so it is disjoint with itself. But then from the axiom of extensionality it follows that the empty set is not identical with itself! But as it turns out, it seems that the axiom of extensionality is formulated in such a way that this contradiction cannot arise. Still, the fact that the empty set has no elements makes it in my view a very troublesome set with no clear identity-conditions. The late philosopher E.J. Lowe argued something similar and concluded that set theory should do away with the empty set. I wouldn't argue that, however, since I'm quite smitten with the set-theoretic derivation of math recursively from the empty set (the Von Neumann approach). Perhaps what I am looking for is a kind of paraconsistent approach to the empty set, where it is precisely the contradiction in the concept of the empty set that will allow us to derive math from it.

In this regard I am also intrigued by Frege's definition of the empty set as the set of all things that are not self-identical. Though I am not sure if from this it follows that the empty set too is not self-identical. After all, the set of all cars is not itself a car. On the other hand, if we define sets extensionally, and we adopt Frege's definition, than the extension of the empty set is not-self-identical, which then seems to imply that the set itself is not-self-identical as well.

Of course, if we start with a contradiction, then ex falso sequitur quodlibet and the entire system will be vitiated. Unless we find a way to somehow contain the contradiction of the empty set (or nothing). My intuition here is that dialectics may be of use here. It seems clear to me that we can say: since nothing is inconsistent, and since being is the negation of nothing, being must be consistent.

I wish I could develop such ideas in a more formal fashion. Perhaps looking into paraconsistent set theory might be of use. If you have any suggestions I would be very much obliged. I am not in any way connected to a university. I got a PhD in philosophy in 2000, but I did not have an academic career. So forums like this one are the only means I have for discussing these things with others and I am sincerely grateful for that!

Also thanks for you paper. I will certainly read it (i.e. I will attempt to read it, since math and formal logic are not my forte, unfortunately). The very idea of a computational approach to neo-platonism certainly seems very original. By the way, I am not such a philosopher who is averse to a scientific approach, quite the contrary, as you may have guessed already. I do think however that presupposing a mathematical reality from which physical somehow derives is still not enough to answer Leibniz' question, for why then is there the mathematical reality to begin with? We can say logic and math are timelessly true, but I think we still want to know why that is so.

This is one thing that can't be explained. It is like asking why is 7 prime? Because no natural numbers besides 1 and 7 evenly divide it. Why not? Well because neither 2, 3, 4, 5, 6, or any number greater than 7 evenly divides it. Why not? That's simply how it is. Personally, I find it very satisfying that a TOE can ultimately have such an intuitive and straight-forward answer to why it is this way and has to be. It's hard to deny 1 != 0.

Jason

 
Moreover: there is also a certain subjectivity involved here, since WE think they are timeless because WE cannot imagine a situation in which logic and math were not true, but then the timelessness is predicated on our cognitive limitations, which does not show that math and logic are in themselves timelessly true.

Peter

--

Bruno Marchal

unread,
Dec 16, 2014, 5:42:32 AM12/16/14
to everyth...@googlegroups.com
On 15 Dec 2014, at 11:22, Alberto G. Corona wrote:

You are projecting metaphisical differences into physical forces at the last steps. That does not make sense IMHO. The New Agers do the opposite.

I think that this is an error typical of people with no education in physics and technology that are overexposed to scientific-tecnical terms.

Your metaphysical reasoning is very interesting. Specially your awareness of the logical positivism and your rejection of it, that is refreshing for me. The people of this list are logical positivists and they don´t know that they adopt this metaphysical standpoint.


I have no clue why you think that we are logical positivist, which in paricular I debunk in detail in many places (forum, papers, books). Then, how could machine's theology fit with logical positism? How could computationalism, which asks for a consciousness invariance act of faith be positivistic? 




I think that the rejection of metaphysics by the logical positivists is an ideological trick that closes their mind and inmmunizes them against metaphysical reasoning, in the same way that marxists despised anything non marxist as bourgeois.

I think that logical positivism, like behaviorisme in psychology has been abandonned by everybody since many decades.

Bruno

Bruno Marchal

unread,
Dec 16, 2014, 12:40:50 PM12/16/14
to everyth...@googlegroups.com
On 17 Nov 2014, at 16:25, Peter Sas wrote:


OK. I print the quite clear and well written introduction of your article:

Peter Sas wrote in his blog:

<<
In my previous post on this blog I argued that if we want to answer Leibniz' famous question ("Why is there something rather than nothing?") we have no choice but to start with the assumption thatnothing at all exists and then investigate how we might derive existence from this state of nothingness. The rationale behind this approach is obvious: as long as we start with some primordial being (e.g. God or the laws of physics) as the cause of all other beings, we will not have truly answered Leibniz' question, since in that case we still have to explain why the supposedly primordial being existed. Why does God exist? Or where did the laws of nature come from? The late Robert Nozick put this problem succinctly as follows: "The question [of Leibniz] appears impossible to answer. Any factor introduced to explain why there is something will itself be part of the something to be explained". (Nozick 1981: 115) Hence, only if we start with the assumption that nothing at all exists will Leibniz' question become answerable.
>>

First, note that you stay in the Aristotelian tradition of suggesting a choice between the two gods of Aristotle: the creator and the creation, that is God or the laws of physics.

Xeusippes (-300), Tegmark and myself and others would add the laws of mathematics. Indeed I show that if we assume that consciousness is invariant for some digital permutation at some level of description, then we cannot distinguishes an arithmetical God from an analytical God nor from a physical God, although we *can* have 3p clues if we attempt to look at ourselves below the computationalist level of description.

Now you will asks me where does the laws of mathematics come from?

Well, computationalism answers that question, not entirely, but it justifies entirely why it doesn't make sense to hope for an entire answer here. It actually isolates the tiniest thing that we can't understand, but need to explain everything.

Let me try to explain. Since the failure of logicism, we know today that we cannot derive the existence of the natural numbers 0, s(0), s(s(0)), ..., or 0, 1, 2, 3, ... if you prefer, from logic alone. So we need axioms, we need a theory, we need hypotheses on which we can hopefully agree to just talk about the natural numbers. Like wize, we can't derive addition from logic, and then even assuming addition, we cannot yet derive multiplication, even with the induction axioms. 

Whatever will be your notion of "nothing", to be enough rich to get the natural number, will make you assuming the natural number, or something Turing equivalent, which is such that if you don't assume it, you cannot get it at all. 

Then, once we have both assumes the addition laws and multiplication laws, we arrive at the Turing universal level, and this contains *all* the machine dreams. Sharable first person plural dreams ("video-game") exists and can cohere to define some multi- or multi-multi-verses. Advantage: the math forces us to distinguish many modalities for the "knowing", "believing", "observing", etc. The person and its consciousness is not eliminated: it is one which put the equations and the fire in the equations. Bute the fundamental equations are the laws of addition and multiplications, all the rest is the pôssible epistemologies of the numbers.

What are you hoping for? The relation between nothing and everything is a relation of complementarity, you can't define one without having the other one. take the unary intersection of sets. That int x = the usual intersection of all sets y belonging to x. Classical logic will make the unary intersection of the empty set equal to the collection of all sets. Or take the number, with multiplication, You get "infinity" when attempting to divide 1 by zero. Or take the quantum emptiness, which assumes often by default some large portion of set theory (much more assumption that elementary arithmetic). But the quantum vacuum contains the universal waves in its partial superposition states, which leads to internal multiverses.

The point is that all precise enough notion of nothingness will assume ... many things, if only at the meta-level. In logic we agree both on the axioms, and of the rules making it possible to derive new formula from those axioms. If you want (but it might be you don't want that) make an explanation of everything from nothing, you will need to make a choice for a notion of "thing", on which we can agree to exist in at least some sense. If that notion is not Turing-universal, you will not be able to explain even just our belief in universal numbers. If it is Turing universal, then it is equivalent to assuming the natural numbers.

In that sense, I think that computationalism does explain (modulo its possible refutation by physics) where everything comes from, including the computable and many things which concern us but are non computable, and some even non nameable.

The only problem, but it is the price of the conceptual solution, and it makes a classical naïve form of computationalism testable, is that we have to derive the laws of physics from arithmetic and number's self-reference abilities, relative to truth, consistency, etc.

Advantage and disadvantage: time to buy some books in math, computer science, mathematical logic.

About -x(Ex)  My problem is that the "reverted E" is a quantifier, and the second E is a property, but I doubt existence can be made into a property. That reify existence, and introduces a problem that we don't need, unless you believe in one or both Aristotelian God(s). Computationalist have not that problem: only 0, s(0) , etc. exist in a clear definite sense. All the rest will be the hallucinations, say, made possible by the relations in between numbers inherited from addition and multiplication. No existence at all is reifed. The basic ontology is 0, s(0), ..., and the physical existence will be when some numbers believe correctly modal proposition of modal existence, like [] <>Ex []<> P(x)  (the first E being the usual quantifier, the physical becomes a point of view on the arithmetical reality "see from inside". There is a bit of:  everything from not a lot: as arithmetic seen from inside is much bigger than arithmetic seen from outside (that's somehow the roots of incompleteness).

Very nice and clear article. I think. But I think that Gödel's theorem, and others by Tarski, Skolem, have shed much light on this question, and computationalism even more, by associating to each (universal) number, a theology, containing physics, so the numbers can evaluate its degree of non-computationalism or emulation-order (not that this easy, but QM could confirm (not prove) that we are NOT in an emulation.

Peter, you are on the territory of philosophy/theology which has a non empty intersection with mathematics, once we assume computationalism.

Bruno
 



Here I use some of the tools of analytical philosophy to analyze the logical impossibility of nothinness... For the philosophically inclined among you...

Peter


--
You received this message because you are subscribed to the Google Groups "Everything List" group.
To unsubscribe from this group and stop receiving emails from it, send an email to everything-li...@googlegroups.com.
To post to this group, send email to everyth...@googlegroups.com.
Visit this group at http://groups.google.com/group/everything-list.
For more options, visit https://groups.google.com/d/optout.

John Clark

unread,
Dec 16, 2014, 12:54:42 PM12/16/14
to everyth...@googlegroups.com
On Mon, Dec 15, 2014 at 12:56 AM, 'Roger' via Everything List <everyth...@googlegroups.com> wrote:

> I propose that a thing exists if it is a grouping or relationship present defining what is contained within.

If nothing is contained within then that is very well defined, therefore nothing exists.  Something obviously also exists, but if both something and nothing exist then there is no contrast and the word "exists" is drained of all usefulness. 

  John K Clark

Roger

unread,
Dec 31, 2014, 11:36:11 PM12/31/14
to everyth...@googlegroups.com
John,

    Hi.  What I was trying to get at is that the most fundamental unit of existence and the most fundamental instantiation of the word exists is the existent entity that is, I think, incorrectly called the "absolute lack-of-all".  That is when you say "therefore nothing exists", what I mean is that this "absolute lack-of-all" is identical to "something".  I'm not  sure how  trying to explain why a thing exists and why "nothing" is actually not the lack of all existent entities but is instead a "something" drains "exists" of any usefulness?
    Thanks.

                    Roger

John Clark

unread,
Jan 1, 2015, 12:17:37 PM1/1/15
to everyth...@googlegroups.com
On Wed, Dec 31, 2014 at 11:36 PM, 'Roger' via Everything List <everyth...@googlegroups.com> wrote:
>>> propose that a thing exists if it is a grouping or relationship present defining what is contained within.

>> If nothing is contained within then that is very well defined, therefore nothing exists.  Something obviously also exists, but if both something and nothing exist then there is no contrast and the word "exists" is drained of all usefulness.

> What I was trying to get at is that the most fundamental unit of existence and the most fundamental instantiation of the word exists is the existent entity that is, I think, incorrectly called the "absolute lack-of-all".

Existent entity? But something that has the existent property is something that exists, and round and round we go. Once again the word "exists" is drained of all usefulness.

 John K Clark


Roger

unread,
Jan 3, 2015, 12:05:46 AM1/3/15
to everyth...@googlegroups.com
Even if the word "exists" has no use because everything exists, it seems important to know why everything exists.  How is it that a thing can exist?  What I suggest is that a grouping defining what is contained within is an existent entity.  Then, you can use this to try and answer the other question of "Why is there something rather than nothing?".

meekerdb

unread,
Jan 3, 2015, 12:44:29 AM1/3/15
to everyth...@googlegroups.com
On 1/2/2015 9:05 PM, 'Roger' via Everything List wrote:
Even if the word "exists" has no use because everything exists, it seems important to know why everything exists.  How is it that a thing can exist?  What I suggest is that a grouping defining what is contained within is an existent entity.  Then, you can use this to try and answer the other question of "Why is there something rather than nothing?".

If everything exists, what doesn't exist?  Nothing.

Brent

Chris de Morsella

unread,
Jan 3, 2015, 1:17:27 AM1/3/15
to everyth...@googlegroups.com

 

 

From: everyth...@googlegroups.com [mailto:everyth...@googlegroups.com] On Behalf Of meekerdb
Sent: Friday, January 02, 2015 9:44 PM
To: everyth...@googlegroups.com
Subject: Re: Why is there something rather than nothing? From quantum theory to dialectics?

 

On 1/2/2015 9:05 PM, 'Roger' via Everything List wrote:

Even if the word "exists" has no use because everything exists, it seems important to know why everything exists.  How is it that a thing can exist?  What I suggest is that a grouping defining what is contained within is an existent entity.  Then, you can use this to try and answer the other question of "Why is there something rather than nothing?".


If everything exists, what doesn't exist?  Nothing.

 

If nothing existed; would it remain nothing?

-Chris

Brent

Alberto G. Corona

unread,
Jan 3, 2015, 8:23:31 AM1/3/15
to everything-list
Logical positivism in the hard form has been abandoned in favor of a dozen derivations, but it is a tactical withdrawal in order to protect the central dogmas: the antimetaphysical standpoint, the acritical adoration of science understood in the very narrow sense of today. The negation of innate knowledge. The negation that the mind can know the truth from inside. The negation of morality as object of study. The negation the most high of man in which distinguish himself form animals.

Or to summarize: the monstruous contradiction of the negation of Man as object of study with the aim to divinize it, Or to be exact, to divinize some men and slave others. That is not possible if morality is objective and the inherent limitations of every men are accepted No men-gods are possible then.

The auto-idealized positivist man look at nature not as a part of it, but as a god that observe nature and submit it to himself trough the knowledge of his laws by science. And this domination include other men. This Man-god justified by himself is the childs treasure that tries to preserve the neo-positivist

Bruno Marchal

unread,
Jan 3, 2015, 10:58:59 AM1/3/15
to everyth...@googlegroups.com
On 03 Jan 2015, at 07:17, 'Chris de Morsella' via Everything List wrote:

 
 
From: everyth...@googlegroups.com [mailto:everyth...@googlegroups.com] On Behalf Of meekerdb
Sent: Friday, January 02, 2015 9:44 PM
To: everyth...@googlegroups.com
Subject: Re: Why is there something rather than nothing? From quantum theory to dialectics?
 
On 1/2/2015 9:05 PM, 'Roger' via Everything List wrote:
Even if the word "exists" has no use because everything exists, it seems important to know why everything exists.  How is it that a thing can exist?  What I suggest is that a grouping defining what is contained within is an existent entity.  Then, you can use this to try and answer the other question of "Why is there something rather than nothing?".

If everything exists, what doesn't exist?  Nothing.
 
If nothing existed; would it remain nothing?

Careful not confusing "Nothing exists" and "Nothing exist".

In the first case, something exists. But not necessarily in the second case. 

Of course not everything exists a priori. There is no divisors of zero different from zero, nor is there a cat-dog, nor is there a triangle with four sides.

Then with mechanism, we can, assume that what exist are simply the numbers 0, s(0), s(s(0)), etc.
Then all the rest, God included, is part of a persistent number hallucination, but "hallucination" should not be used as "unreal", because the hallucination is real, and is what makes our lives, and there is no reason to dismiss them at all. 

The math makes this clear too by distinguish the 

ontical existence  Ex P(x)  and only 0, s(0), ... exists in that sense

and the many and quite variate rich phenomenological existence: whcih are obtained with the modal points of view, like []Ex[]P(x), with [] being the box of self-reference logic and its many intensional variants (which distinguish basicall all science (biology, psychology, physics, even theology).


Bruno









-Chris

Brent
-- 
You received this message because you are subscribed to the Google Groups "Everything List" group.
To unsubscribe from this group and stop receiving emails from it, send an email to everything-li...@googlegroups.com.
To post to this group, send email to everyth...@googlegroups.com.
Visit this group at http://groups.google.com/group/everything-list.
For more options, visit https://groups.google.com/d/optout.

-- 
You received this message because you are subscribed to the Google Groups "Everything List" group.
To unsubscribe from this group and stop receiving emails from it, send an email to everything-li...@googlegroups.com.
To post to this group, send email to everyth...@googlegroups.com.
Visit this group at http://groups.google.com/group/everything-list.
For more options, visit https://groups.google.com/d/optout.

Roger

unread,
Jan 4, 2015, 2:07:18 AM1/4/15
to everyth...@googlegroups.com
In regard to:


"If nothing existed; would it remain nothing?
"

This is exactly what I'm suggesting.  It would not remain "nothing".  We usually think of the situation when you get rid of all matter, energy, space/volume, time, abstract concepts, minds, etc. as "nothing".  But, what I'm saying is that this supposed "nothing" really isn't the lack of all existent entities.  That "nothing" would be the entirety of all that is present; that's it; there's nothing else.  It would be the all.  An entirety is a grouping defining what is contained within and therefore an existent entity, based on my definition of an existent entity.   So, even what we think of as "nothing" is an existent entity or "something".  This means that "something" is non-contingent.  It's necessary.  There is no such thing as the lack of all existent entities.

Chris de Morsella

unread,
Jan 4, 2015, 3:30:04 AM1/4/15
to everyth...@googlegroups.com

 

 

From: everyth...@googlegroups.com [mailto:everyth...@googlegroups.com] On Behalf Of Bruno Marchal
Sent: Saturday, January 03, 2015 7:59 AM
To: everyth...@googlegroups.com
Subject: Re: Why is there something rather than nothing? From quantum theory to dialectics?

 

 

On 03 Jan 2015, at 07:17, 'Chris de Morsella' via Everything List wrote:



 

 

From: everyth...@googlegroups.com [mailto:everyth...@googlegroups.com] On Behalf Of meekerdb
Sent: Friday, January 02, 2015 9:44 PM
To: everyth...@googlegroups.com
Subject: Re: Why is there something rather than nothing? From quantum theory to dialectics?

 

On 1/2/2015 9:05 PM, 'Roger' via Everything List wrote:

Even if the word "exists" has no use because everything exists, it seems important to know why everything exists.  How is it that a thing can exist?  What I suggest is that a grouping defining what is contained within is an existent entity.  Then, you can use this to try and answer the other question of "Why is there something rather than nothing?".


If everything exists, what doesn't exist?  Nothing.

 

If nothing existed; would it remain nothing?

 

Careful not confusing "Nothing exists" and "Nothing exist".

 

In the first case, something exists. But not necessarily in the second case. 

 

Okay… I see you point. “Nothing Exist” is a hard abstraction to wrap the mind around and the mind will try like hell to give nothing a kind of existence because it is so impossibly hard to even imagine the former.

 

Of course not everything exists a priori. There is no divisors of zero different from zero,

 

>>nor is there a cat-dog,

 

Not yet in our universe, but what about in fifty years from now would it remain beyond our technical reach to fuse the DNA of a cat and a dog to create this radical hybrid? Would it always fight with itself… would it bark or meow J

I take your point however.

 

nor is there a triangle with four sides.

 

Then with mechanism, we can, assume that what exist are simply the numbers 0, s(0), s(s(0)), etc.

 

I don’t think you are referring to set notation.. the empty set being {}. So by “s(0)” do you mean an operation taking zero? A specific operation perhaps: 0, sum(0), sum(sum(0)) etc. ?

It seems so but I am not sure.

 

Then all the rest, God included, is part of a persistent number hallucination, but "hallucination" should not be used as "unreal", because the hallucination is real, and is what makes our lives, and there is no reason to dismiss them at all. 

 

The math makes this clear too by distinguish the 

 

ontical existence  Ex P(x)  and only 0, s(0), ... exists in that sense

 

and the many and quite variate rich phenomenological existence: whcih are obtained with the modal points of view, like []Ex[]P(x), with [] being the box of self-reference logic and its many intensional variants (which distinguish basicall all science (biology, psychology, physics, even theology).

 

It is intuitive to me how a vastly deep self-referential recursion of math could generate all manner of sublime subtle effects at some far remove from the basic fundamental math underlying the self-referential edifice.

-Chris

 

Bruno

 

 

 

 

 

 

 

 



-Chris

Brent

-- 
You received this message because you are subscribed to the Google Groups "Everything List" group.
To unsubscribe from this group and stop receiving emails from it, send an email to everything-li...@googlegroups.com.
To post to this group, send email to everyth...@googlegroups.com.
Visit this group at http://groups.google.com/group/everything-list.
For more options, visit https://groups.google.com/d/optout.

 

-- 
You received this message because you are subscribed to the Google Groups "Everything List" group.
To unsubscribe from this group and stop receiving emails from it, send an email to everything-li...@googlegroups.com.
To post to this group, send email to everyth...@googlegroups.com.
Visit this group at http://groups.google.com/group/everything-list.
For more options, visit https://groups.google.com/d/optout.

Chris de Morsella

unread,
Jan 4, 2015, 3:49:24 AM1/4/15
to everyth...@googlegroups.com

In regard to:

"If nothing existed; would it remain nothing?
"

This is exactly what I'm suggesting.  It would not remain "nothing".  We usually think of the situation when you get rid of all matter, energy, space/volume, time, abstract concepts, minds, etc. as "nothing".  But, what I'm saying is that this supposed "nothing" really isn't the lack of all existent entities.  That "nothing" would be the entirety of all that is present; that's it; there's nothing else.  It would be the all.  An entirety is a grouping defining what is contained within and therefore an existent entity, based on my definition of an existent entity.   So, even what we think of as "nothing" is an existent entity or "something".  This means that "something" is non-contingent.  It's necessary.  There is no such thing as the lack of all existent entities.

 

Roger – you have much to say about nothing [just joking]

I agree with the distinction you make between nothing arrived at through the negative process of removing everything that exists until nothing is left versus the nothing *that is* everything.

Further down, if I follow you, you are making the point that if we are speaking about the *nothing that is the set of everything there is* then even if this is an empty set, by virtue of a set being something – a conceptual entity – then even the absolutely empty universal set {} exists as a conceptual entity at least.

Is that a fair recap of your intent; or am I off the mark?

-Chris



On Saturday, January 3, 2015 1:17:27 AM UTC-5, cdemorsella wrote:

 

 

From: everyth...@googlegroups.com [mailto:everyth...@googlegroups.com] On Behalf Of meekerdb
Sent: Friday, January 02, 2015 9:44 PM
To: everyth...@googlegroups.com
Subject: Re: Why is there something rather than nothing? From quantum theory to dialectics?

 

On 1/2/2015 9:05 PM, 'Roger' via Everything List wrote:

Even if the word "exists" has no use because everything exists, it seems important to know why everything exists.  How is it that a thing can exist?  What I suggest is that a grouping defining what is contained within is an existent entity.  Then, you can use this to try and answer the other question of "Why is there something rather than nothing?".


If everything exists, what doesn't exist?  Nothing.

 

If nothing existed; would it remain nothing?

-Chris

Brent

--
You received this message because you are subscribed to the Google Groups "Everything List" group.
To unsubscribe from this group and stop receiving emails from it, send an email to everything-li...@googlegroups.com.
To post to this group, send email to everyth...@googlegroups.com.
Visit this group at http://groups.google.com/group/everything-list.
For more options, visit https://groups.google.com/d/optout.

Kim Jones

unread,
Jan 4, 2015, 4:09:15 AM1/4/15
to everyth...@googlegroups.com



 

On 3 Jan 2015, at 5:17 pm, 'Chris de Morsella' via Everything List <everyth...@googlegroups.com> wrote:

If everything exists, what doesn't exist?  Nothing.

 

If nothing existed; would it remain nothing?

-Chris

Brent


You are both missing the main question: what was there before there was nothing?

K

John Clark

unread,
Jan 4, 2015, 12:38:17 PM1/4/15
to everyth...@googlegroups.com
On Sat, Jan 3, 2015  'Roger' via Everything List <everyth...@googlegroups.com> wrote:

> Even if the word "exists" has no use because everything exists, it seems important to know why everything exists.
 
Even if the word "klogknee" has no use because everything is klogknee, is it important to know why everything is klogknee?
 
> How is it that a thing can exist? 
 
How is it that a thing can be klogknee? Before you can figure that out you must first know what "klogknee" means, and if everything is klogknee then you don't know because meaning needs contrast. 

> What I suggest is that a grouping defining what is contained within is an existent entity. 

I don't understand that.

  John K Clark




John Clark

unread,
Jan 4, 2015, 1:31:40 PM1/4/15
to everyth...@googlegroups.com
On Sat, Jan 3, 2015 at 10:58 AM, Bruno Marchal <mar...@ulb.ac.be> wrote:


> Careful not confusing "Nothing exists" and "Nothing exist". In the first case, something exists. But not necessarily in the second case


If "nothing" means no-thing, and that is certainly how that English word originated, then the meaning of the first case is clear even if I don't agree with what it says, but "no thing exist" just sounds like bad grammar to me. 

  John K Clark





 


 


meekerdb

unread,
Jan 4, 2015, 1:41:34 PM1/4/15
to everyth...@googlegroups.com
When is there something?  Now!  When wasn't there something?
Never!
    --- with apologies to W. V. O. Quine

Chris de Morsella

unread,
Jan 4, 2015, 2:26:27 PM1/4/15
to everyth...@googlegroups.com

 

 

From: everyth...@googlegroups.com [mailto:everyth...@googlegroups.com] On Behalf Of Kim Jones
Sent: Sunday, January 04, 2015 1:09 AM
To: everyth...@googlegroups.com
Subject: Re: Why is there something rather than nothing? From quantum theory to dialectics?

 

 

 

 

Why, pre-nothing, of course.

-Chris

 

K

LizR

unread,
Jan 4, 2015, 2:27:16 PM1/4/15
to everyth...@googlegroups.com
I hope Russell's theory of nothing is getting due attention.

Chris de Morsella

unread,
Jan 4, 2015, 5:10:57 PM1/4/15
to everyth...@googlegroups.com

 

 

From: everyth...@googlegroups.com [mailto:everyth...@googlegroups.com] On Behalf Of LizR

 

I hope Russell's theory of nothing is getting due attention.

 

Russell’s observation that “The ultimate theory of everything is just a theory of nothing.” seems intuitively correct to me… though I have no rigorous proof for this sense of it ringing true for me.

 

I was in ignorance that Russell had written a book on this; and just downloaded the pdf – so thanks Liz for bringing it to attention. Beginning to read it now….

Another excellent passage: “Something is the “inside view” of Nothing”. Nice! And this view from the inside looks so infinitely full of all manner of emergent stuff. I agree with the premise that perspective is paramount in coming to terms with and to understand the spooky weird nature of quantum reality; perspective also provides a powerful tool to explain the “something from nothing paradox”. Something does seem like it could be how Nothing looks from the perspective of being within itself – as opposed to the bird’s eye view from outside -- Max Tegmark uses Bird’s Eye view to describe this outside privileged perspective… looking down on the examined system from an outside perspective (even if that system, is everything that is… it is still valuable as an intellectual tool to be able to view this from the outside perspective as well).. but I digress, back to the book.

 

One question for Russell, wonder what his thoughts are on the continued viability of Quantum Loop Gravity hypothesis – which you mention as being one of the contenders along with String Theory – for the unification of all the fundamental forces into a single theory -- given the findings of the ESA experiment that has showed that spacetime must be smooth down to scales trillions of times smaller than the Planck scale.

 

[If I recall they differentially measured the polarity of light from a distant and very powerful gamma ray burst over many wavelengths, from the hard gamma rays down through other wavelengths of light issuing from the same phenomenon. Their argument is that if space time was granular then this would have interacted with the passing light and induced a polarity bias that would affect different wave lengths of light differently. This is a I recall the details of the experiment. What they found instead is a lack of any effect – down to the incredibly small sub-Planck scale they were able to indirectly peer down into)]

 

Doesn’t Quantum Loop Gravity require space time to be granular at the Planck scale? And if so isn’t the ESA experimental evidence a potential falsification of the hypothesis – at least as it has been formulated?

 

“Thus we should conclude the opposite of what we first supposed. Far from containing the wisdom of the ages, the library is useless, containing no information of worth. Our libraries are useful, not so much for the books they contain, but for the books they don’t contain!”

 

I detect an echo of the Tao Te Ching by Lao Tzu in this statement. “The thirty spokes unite in the one nave; but it is on the empty space (for the axle), that the use of the wheel depends. Clay is fashioned into vessels; but it is on their empty hollowness, that their use depends. The door and windows are cut out (from the walls) to form an apartment; but it is on the empty space (within), that its use depends.

 

And so… by the same Daoist token, it is the absence of information that makes any given collection of information useful.

 

“The validity of the anthropic principle tells us that self-awareness must somehow be necessary to consciousness.” I agree with that; it is only by reflecting on the self and being aware of the self-nature we are observing that we can become conscious of its existence…. Of our existence.

 

“all laws of physics will eventually be found to relate back to some essential property of the conscious observer” – the fundamental centrality of the observer for understanding reality is an idea I have long found intriguing.

 

Excellent intro Russell.. I now know what I will be spending my Sunday afternoon (and maybe evening on). On to the next chapter.

 

Cheers,

-Chris

Chris de Morsella

unread,
Jan 4, 2015, 5:17:46 PM1/4/15
to everyth...@googlegroups.com

Russell ~ got to say that you nailed it on the head, with this statement: “Thus it appears that emergence stands in opposition to reductionism, a paradigm of understanding something by studying its constituent parts. To someone wedded to the notion of reductionism, emergence can appear rather mysterious and strange.”

Roger

unread,
Jan 4, 2015, 11:47:18 PM1/4/15
to everyth...@googlegroups.com
Chris,

       I have nothing important to say! :-)  Nothing and something are kind of good areas for puns, double entendres and jokes.  After all, Jerry Seinfeld had a whole show about nothing!
>Roger – you have much to say about nothing [just joking]

    You mentioned:
>I agree with the distinction you make between nothing arrived at through the negative process of removing everything that >exists until nothing is left versus the nothing *that is* everything.

>Further down, if I follow you, you are making the point that if we are speaking about the *nothing that is the set of >everything there is* then even if this is an empty set, by virtue of a set being something – a conceptual entity – then even >the absolutely empty universal set {} exists as a conceptual entity at least.

>Is that a fair recap of your intent; or am I off the mark?

    I think that's a good recap of my intent.  If we can visualize the "absolute lack-of-all" where all things traditionally thought to exist, including our minds doing the imagining, that nothingness would be everything there is.  And, then as you say, I think everything there is is a grouping defining what is contained within and therefore an existent entity.  A set is also a grouping defining what is contained within, so this situation would be similar to the empty set.  I think this fundamental existent entity similar to the empty set is the fundamental unit of our physical universe.

    Also, you mentioned in a later post: 

>Something is the “inside view” of Nothing”....I agree with the premise that perspective is paramount in coming to terms with >and to understand the spooky weird nature of quantum reality; perspective also provides a powerful tool to explain the >“something from nothing paradox”. Something does seem like it could be how Nothing looks from the perspective of being >within itself – as opposed to the bird’s eye view from outside 

    I totally agree that perspective is paramount in deciding whether the "absolute lack-of-all" is "something" or "nothing".  But, I always like to think that when we're inside "nothingness", that means we're also like "nothingness", so this "nothingness" just looks like "nothing".  But, if we could step outside that "nothingness", we'd see that it is the entirety of all there is and thus an existent entity.

    In regard to Russell's stuff on nothingness, I can't remember the details now, but I think I read about it at one time and don't remember its really answering any questions. 

    Have a good week!

                                               Roger  

Russell Standish

unread,
Jan 5, 2015, 2:29:27 AM1/5/15
to everyth...@googlegroups.com
On Sun, Jan 04, 2015 at 02:10:51PM -0800, 'Chris de Morsella' via Everything List wrote:
>
>
>
> Russell’s observation that “The ultimate theory of everything is just a theory of nothing.” seems intuitively correct to me… though I have no rigorous proof for this sense of it ringing true for me.
>
>
>
> I was in ignorance that Russell had written a book on this; and just downloaded the pdf – so thanks Liz for bringing it to attention. Beginning to read it now….
>
> Another excellent passage: “Something is the “inside view” of Nothing”. Nice! And this view from the inside looks so infinitely full of all manner of emergent stuff. I agree with the premise that perspective is paramount in coming to terms with and to understand the spooky weird nature of quantum reality; perspective also provides a powerful tool to explain the “something from nothing paradox”. Something does seem like it could be how Nothing looks from the perspective of being within itself – as opposed to the bird’s eye view from outside -- Max Tegmark uses Bird’s Eye view to describe this outside privileged perspective… looking down on the examined system from an outside perspective (even if that system, is everything that is… it is still valuable as an intellectual tool to be able to view this from the outside perspective as well).. but I digress, back to the book.
>
>
>
> One question for Russell, wonder what his thoughts are on the continued viability of Quantum Loop Gravity hypothesis – which you mention as being one of the contenders along with String Theory – for the unification of all the fundamental forces into a single theory -- given the findings of the ESA experiment that has showed that spacetime must be smooth down to scales trillions of times smaller than the Planck scale.
>

Thanks for your kind words. Actually as to whether loop gravity or
string theory or something else is the way to go, I really don't have
a dog in the fight. I was merely commenting on my confidence that
gravity will ultimately be unified with electro-weak-strong forces in
some manner, but being agnostic as to how.

My personal opinion is that measured values are constrained to be
rational - there can only ever be a countable number of distinct
observer moments. Yet this down not imply space is "quantised" or
discrete in any way. It is quite possible there is no lower bound to
the difference between two measurements. So it doesn't surprise me
that space ends up being smooth at scales far smaller than the planck
length. I would be more suprised at the opposite conclusion, as it
implies a lack of symmetry (grids are not rotationally symmetric,
except at specific angles).

As for unification of GR and QM, one wildly speculative thought I've
had is that matter is due to knots in space-time, and that the
different types of particle relate to the different types of knots
possible in a 4D Riemannian manifold. Some knots are easier to undo
than others, explaining different particle lifetimes. Mass appears as
curvature of space, so the knots have mass due to the twist they
impart on spacetime. But importantly, matter does not "curve
spacetime", as is typically said, but matter is more of a topological defect.

I have no idea if this idea has legs - I don't currently have the
mathematical chops to work it through, and unfortunately also insufficient
interest to acquire the necessary mathematical skills.


--

----------------------------------------------------------------------------
Prof Russell Standish Phone 0425 253119 (mobile)
Principal, High Performance Coders
Visiting Professor of Mathematics hpc...@hpcoders.com.au
University of New South Wales http://www.hpcoders.com.au

Latest project: The Amoeba's Secret
(http://www.hpcoders.com.au/AmoebasSecret.html)
----------------------------------------------------------------------------

Chris de Morsella

unread,
Jan 5, 2015, 4:07:29 AM1/5/15
to everyth...@googlegroups.com

 

 

From: everyth...@googlegroups.com [mailto:everyth...@googlegroups.com]
Sent: Sunday, January 04, 2015 8:47 PM

 

Chris,

 

       I have nothing important to say! :-)  Nothing and something are kind of good areas for puns, double entendres and jokes.  After all, Jerry Seinfeld had a whole show about nothing!

>Roger – you have much to say about nothing [just joking]

 

Kind of good?

Roger, it’s a friggen gold mine; never figured it out ;)  but people love to laugh about nothing J

More than one adept has said that laughter about nothing can be the key that opens the way to everything!

And, there is nothing wrong with laughing about nothing, cause nothing cares :)

 

 

    You mentioned:

>I agree with the distinction you make between nothing arrived at through the negative process of removing everything that >exists until nothing is left versus the nothing *that is* everything.

>Further down, if I follow you, you are making the point that if we are speaking about the *nothing that is the set of >everything there is* then even if this is an empty set, by virtue of a set being something – a conceptual entity – then even >the absolutely empty universal set {} exists as a conceptual entity at least.

>Is that a fair recap of your intent; or am I off the mark?

>>I think that's a good recap of my intent.  If we can visualize the "absolute lack-of-all" where all things traditionally thought to exist, including our minds doing the imagining, that nothingness would be everything there is.  

 

As a thought experiment it is hard to put oneself into the perspective of imagining the *absolute lack-of-all* because, in this case, the observer doing the observing is also paradoxically non-existent. And in the case of one’s own self, especially, achieving this perspective is devilishly hard thing to do… the self always strains to creep back in from some corner or hidden crevice of the contemplating mind. There is a lot of value -- in my experience -- from making an honest (with oneself) good faith attempt… it is like riding a unicycle backwards on your hands, instead of your feet… so not an easy trick to master, but my feeling is that the attempt is worthy and may (in a ready brain) possibly be mind-expanding. Identity, after all, can become its own prison, and laughter about nothing can sometimes lead to a realization of nothing, which is something like nothing else can ever be and is a perfect antidote to the identity doldrums!

 

but I digress.

 

>>And, then as you say, I think everything there is is a grouping defining what is contained within and therefore an existent entity.  A set is also a grouping defining what is contained within, so this situation would be similar to the empty set.  I think this fundamental existent entity similar to the empty set is the fundamental unit of our physical universe.

 

The empty set itself – especially when viewed as a dynamic computational entity could provide that first spark to jump start the engine of everything… so essentially agreeing with you on this view.

It also has an auto-catalytic nature that makes it appealing for me as well, as being the prime first mover of everything from nothing. The set is a priori defined, by being the container of nothing; there is no need to invent it e.g. it auto-catalyzes itself because it is nothing

This is the specific part I still have some problems with, because though the set is a pure conceptual entity, it never the less is also imbued with a rich set of operations and properties. Even the empty set is a non-trivial conceptual entity.

Part of me says that one cannot exclude that there may exist some even more fundamental base conceptual entity (itself also naturally auto-emergent from nothing) from which the set itself is emergent.

In pure nothing – a different beast as we both agree from the notion of what is left when everything is removed – the container e.g. the set only can exist if nothing gains perspective on itself. The set arises naturally from nothing because it is a priori the container, but perhaps only because perspective arose as the auto-catalyzing agent to enable nothing to obtain self-perspective and self-realize it is contained… and then from the concept of the container to set operations.

This is where I feel the need for a basic root set of some simple arithmetic operations, summation etc. to enable longer, and increasingly self-referential as well as recursive equations. Multiplication can be expressed in terms of summation, subtraction in terms of summation plus negation (e.g. negative summation), division in terms of subtraction, termination predicate and an accumulator… and so on.

With just the two simple operations of summation and negation plus the implicit operation of equation (‘=’), a rich set of follow on emergent operations can be derived. Combining these simple operations with the container things can start to get interesting

Maybe a series like this:

0

0=0 leading to

0={0} and then onward to: 0={0}= {0}+{0} = {{0}, {0}+{0}} etc.

Or as Bruno put it: 0, s(0), s(s(0)),

Very similar, in my view, except more focused on the concept of the basic minimal set of operations (if I understand his view)

I am reminded of Bruno’s rather poetic description, on an earlier thread… speaking about the emergence of reality from this pure nothing out of simple self-referential and infinitely recursive summation  “Then all the rest, God included, is part of a persistent number hallucination, but "hallucination" should not be used as "unreal", because the hallucination is real, and is what makes our lives, and there is no reason to dismiss them at all. 

 

Beautifully put IMO… chasing down the lair where hides this hallucination is the greatest hunt of them all. Well at least for the rare bird that sees something worth hunting in nothing.

 

 

    Also, you mentioned in a later post: 

 

>Something is the “inside view” of Nothing”....I agree with the premise that perspective is paramount in coming to terms with >and to understand the spooky weird nature of quantum reality; perspective also provides a powerful tool to explain the >“something from nothing paradox”. Something does seem like it could be how Nothing looks from the perspective of being >within itself – as opposed to the bird’s eye view from outside 

 

    I totally agree that perspective is paramount in deciding whether the "absolute lack-of-all" is "something" or "nothing".  But, I always like to think that when we're inside "nothingness", that means we're also like "nothingness", so this "nothingness" just looks like "nothing".  But, if we could step outside that "nothingness", we'd see that it is the entirety of all there is and thus an existent entity.

 

That is a lovely degree of abstraction J

Again – it would seem – perspective is key. For as you point out when we adopt the inside perspective we become the inside perspective AND all that goes along with it! Could there exist a subtle self-referential perspective of having the outside perspective on adopting the inside perspective… I wonder.

Why do I ask, because this seems critical, because if the inside perspective remains sealed and cutoff from ever being able to have the inside perspective and vice a versa how can they ever communicate or have knowledge about each other. I see the need for the special perspective that has a foot – so to speak – in both the inside view of nothing and the outside view that is achieved by the container of nothing (i.e. the empty set)

Ending this particular response -- right now -- before my brain explodes on nothing

Cheers

-Chris

Chris de Morsella

unread,
Jan 5, 2015, 5:04:07 AM1/5/15
to everyth...@googlegroups.com


-----Original Message-----
From: everyth...@googlegroups.com [mailto:everyth...@googlegroups.com] On Behalf Of Russell Standish
Sent: Sunday, January 04, 2015 11:38 PM
To: everyth...@googlegroups.com
Subject: Re: Why is there something rather than nothing? From quantum theory to dialectics?

On Sun, Jan 04, 2015 at 02:10:51PM -0800, 'Chris de Morsella' via Everything List wrote:
>
>
>
> Russell’s observation that “The ultimate theory of everything is just a theory of nothing.” seems intuitively correct to me… though I have no rigorous proof for this sense of it ringing true for me.
>
>
>
> I was in ignorance that Russell had written a book on this; and just downloaded the pdf – so thanks Liz for bringing it to attention. Beginning to read it now….
>
> Another excellent passage: “Something is the “inside view” of Nothing”. Nice! And this view from the inside looks so infinitely full of all manner of emergent stuff. I agree with the premise that perspective is paramount in coming to terms with and to understand the spooky weird nature of quantum reality; perspective also provides a powerful tool to explain the “something from nothing paradox”. Something does seem like it could be how Nothing looks from the perspective of being within itself – as opposed to the bird’s eye view from outside -- Max Tegmark uses Bird’s Eye view to describe this outside privileged perspective… looking down on the examined system from an outside perspective (even if that system, is everything that is… it is still valuable as an intellectual tool to be able to view this from the outside perspective as well).. but I digress, back to the book.
>
>
>
> One question for Russell, wonder what his thoughts are on the continued viability of Quantum Loop Gravity hypothesis – which you mention as being one of the contenders along with String Theory – for the unification of all the fundamental forces into a single theory -- given the findings of the ESA experiment that has showed that spacetime must be smooth down to scales trillions of times smaller than the Planck scale.
>

Thanks for your kind words. Actually as to whether loop gravity or string theory or something else is the way to go, I really don't have a dog in the fight. I was merely commenting on my confidence that gravity will ultimately be unified with electro-weak-strong forces in some manner, but being agnostic as to how.

My personal opinion is that measured values are constrained to be rational - there can only ever be a countable number of distinct observer moments. Yet this down not imply space is "quantised" or discrete in any way. It is quite possible there is no lower bound to the difference between two measurements. So it doesn't surprise me that space ends up being smooth at scales far smaller than the planck length. I would be more suprised at the opposite conclusion, as it implies a lack of symmetry (grids are not rotationally symmetric, except at specific angles).

As for unification of GR and QM, one wildly speculative thought I've had is that matter is due to knots in space-time, and that the different types of particle relate to the different types of knots possible in a 4D Riemannian manifold. Some knots are easier to undo than others, explaining different particle lifetimes. Mass appears as curvature of space, so the knots have mass due to the twist they impart on spacetime. But importantly, matter does not "curve spacetime", as is typically said, but matter is more of a topological defect.

I have no idea if this idea has legs - I don't currently have the mathematical chops to work it through, and unfortunately also insufficient interest to acquire the necessary mathematical skills.

Nice... I like where you are going with that!
I will need to sleep on that for it is late here -- now -- in my particular spot on earth, deep in the night shadow... though, the long trail of earth's shadow, this ephemeral umbilical cord to the universe beckons me to stay awake... my brain, frankly, says otherwise.
Want to think about that some more (as Monday permits) it sounds like an intriguing mathematical, reality making mechanism.
-Chris
P.S. Also to respond to your other points, but now the sand man has come for me.



--

----------------------------------------------------------------------------
Prof Russell Standish Phone 0425 253119 (mobile)
Principal, High Performance Coders
Visiting Professor of Mathematics hpc...@hpcoders.com.au
University of New South Wales http://www.hpcoders.com.au

Latest project: The Amoeba's Secret
(http://www.hpcoders.com.au/AmoebasSecret.html)
----------------------------------------------------------------------------

John Clark

unread,
Jan 5, 2015, 1:44:48 PM1/5/15
to everyth...@googlegroups.com
On Mon, Jan 5, 2015  Russell Standish <li...@hpcoders.com.au> wrote:

> My personal opinion is that measured values are constrained to be rational

If that is true (and it may be) and if mathematics is a language and the irrational numbers play no role in physics then perhaps they are a fantasy, the equivalent of a Harry Potter novel. Or perhaps not, there are lots of ways to write a sequel to a Harry Potter novel but only one way a new story about the irrationals could go. A story about the irrational numbers is unique, a story about Harry Potter is not, would that be enough to say the irrationals are not a fantasy? I don't know.     

> there can only ever be a countable number of distinct observer moments.

I wonder if countable is good enough, I wonder if it must also be finite.

> Yet this down not imply space is "quantised" or discrete in any way. It is quite possible there is no lower bound to the difference between two measurements. So it doesn't surprise me that space ends up being smooth at scales far smaller than the planck length. I would be more suprised at the opposite conclusion, as it implies a lack of symmetry (grids are not rotationally symmetric, except at specific angles).

I don't think that by itself would necessarily be a deal breaker, electric, magnetic and gravitational fields need not be rotationally symmetric. And lots of interesting stuff happens when symmetry is broken, water is rotationally symmetric but ice is not, and most (but not all) lifeforms on this planet have no rotational symmetry.

  John K Clark
 

meekerdb

unread,
Jan 5, 2015, 1:54:56 PM1/5/15
to everyth...@googlegroups.com
On 1/5/2015 1:07 AM, 'Chris de Morsella' via Everything List wrote:
0={0} and then onward to: 0={0}= {0}+{0} = {{0}, {0}+{0}} etc.

There's your problem: "etc"

Brent

Chris de Morsella

unread,
Jan 5, 2015, 2:24:44 PM1/5/15
to everyth...@googlegroups.com

 

 

From: everyth...@googlegroups.com [mailto:everyth...@googlegroups.com] On Behalf Of meekerdb
Sent: Monday, January 05, 2015 10:55 AM
To: everyth...@googlegroups.com
Subject: Re: Why is there something rather than nothing? From quantum theory to dialectics?

 

On 1/5/2015 1:07 AM, 'Chris de Morsella' via Everything List wrote:

0={0} and then onward to: 0={0}= {0}+{0} = {{0}, {0}+{0}} etc.


There's your problem: "etc"

 

Not trying to go all the way – really trying to chase down some hypothetical candidate for prime mover. What could have kicked it off – from a hypothesized nothing that was before time and anything else. Assuming, for the sake of discussion that there was a theory of everything, out of nothing, then what do you think the very first prime agent or entity might be that could get everything going from nothing at all… from the nothing that encompasses everything?

Is it because you feel that pure mathematical structures are an insufficient foundation for reality? This is a commonly held view by many physicists (very likely the majority)

Or is it because you don’t think that sets plus simple summation, negation and equivalence operations along with the property of perspective is sufficient. Can the concept of a countable series arise out from sets or do numbers at least the first number one and then all others derived through the application of summation to create a new number entity (e.g. the sum of the inputs) each time.

Not professing that I know any answers… but I must admit I find this to be a fascinating problem.

meekerdb

unread,
Jan 5, 2015, 4:15:16 PM1/5/15
to everyth...@googlegroups.com
On 1/5/2015 11:24 AM, 'Chris de Morsella' via Everything List wrote:

 

 

From: everyth...@googlegroups.com [mailto:everyth...@googlegroups.com] On Behalf Of meekerdb
Sent: Monday, January 05, 2015 10:55 AM
To: everyth...@googlegroups.com
Subject: Re: Why is there something rather than nothing? From quantum theory to dialectics?

 

On 1/5/2015 1:07 AM, 'Chris de Morsella' via Everything List wrote:

0={0} and then onward to: 0={0}= {0}+{0} = {{0}, {0}+{0}} etc.


There's your problem: "etc"

 

Not trying to go all the way – really trying to chase down some hypothetical candidate for prime mover. What could have kicked it off – from a hypothesized nothing that was before time and anything else. Assuming, for the sake of discussion that there was a theory of everything, out of nothing, then what do you think the very first prime agent or entity might be that could get everything going from nothing at all… from the nothing that encompasses everything?


All that assumes that nothing is some natural prior state from which everything arose.  I think that's a myth left over from creator god myths.  Cosmologically a better model may be eternal inflation.


Is it because you feel that pure mathematical structures are an insufficient foundation for reality? This is a commonly held view by many physicists (very likely the majority)


I think mathematics and logic are inventions of human language.  They may be necessary in the sense that any intelligent race would invent the same thing - but they might not.  Consider the discussions of the foundations of mathematics, such as this one I just ran across:
https://drive.google.com/file/d/0BwBB5W78Z2I3ZkFNVDhmOXpHQVk/view?pli=1
I see two problems with taking mathematics as fundamental. First, "mathematics" isn't a single thing.  There are different "mathematics" which are contradictory; so effectively you have to chose one and say "This is the mathematics that really exists." (Bruno choses arithmetic or its equivalence).  But this raises the same kind of question about what "exist" means as raised by saying matter exists. Second, the axiomatic nature of mathematics means that you can only get out of it what is already implicit in the axioms and the rules of inference.  Choose different axioms and you get different theories.



Or is it because you don’t think that sets plus simple summation, negation and equivalence operations along with the property of perspective is sufficient.


In some sense it must be "sufficient" since it is an infinite structure and it must be capable of modeling all possible human experience (which is finite).  But that's a trivial sense of "sufficient".  It's like the everything that equals nothing, i.e. no information.  That's why I emphasize that the test of a good theory isn't explanation, it's prediction.

Brent

LizR

unread,
Jan 5, 2015, 6:50:08 PM1/5/15
to everyth...@googlegroups.com
Eternal inflation seems to assume there is something because "there has always been something". However if so, it sidesteps the underlying issue - why is there this (eternal) something? The question itself - and any attempted answer - can't be answered causally.

meekerdb

unread,
Jan 5, 2015, 7:35:15 PM1/5/15
to everyth...@googlegroups.com
On 1/5/2015 3:50 PM, LizR wrote:
Eternal inflation seems to assume there is something because "there has always been something". However if so, it sidesteps the underlying issue - why is there this (eternal) something? The question itself - and any attempted answer - can't be answered causally.

"Sidesteps"? or shows it's an invented issue.  If there were nothing would the issue be why isn't there something?  Why should nothing be unquestionably accepted as the default that needs no explanation?

Brent

Chris de Morsella

unread,
Jan 5, 2015, 7:43:54 PM1/5/15
to everyth...@googlegroups.com


From: meekerdb <meek...@verizon.net>
To: everyth...@googlegroups.com
Sent: Monday, January 5, 2015 4:34 PM

Subject: Re: Why is there something rather than nothing? From quantum theory to dialectics?
On 1/5/2015 3:50 PM, LizR wrote:
Eternal inflation seems to assume there is something because "there has always been something". However if so, it sidesteps the underlying issue - why is there this (eternal) something? The question itself - and any attempted answer - can't be answered causally.

"Sidesteps"? or shows it's an invented issue.  If there were nothing would the issue be why isn't there something?  Why should nothing be unquestionably accepted as the default that needs no explanation?

Nice question. 
No easy answer jumps out as being the obvious answer either. Why the human mind seems to come up with this assumption over and over again across various cultures and periods of history. Why do you think humans seem to accept this by and large as the default base state of everything. Could it be an artifact of the way our minds work? 
The idea of nothing as the default base state of the universe -- before God (or Nature) created everything - seems quite widespread.

LizR

unread,
Jan 5, 2015, 8:09:09 PM1/5/15
to everyth...@googlegroups.com
Sounds a bit like sophistry. The question could be rephrased 'why/how does anything exist?' - which is a natural continuation of the questions scientists have been asking for a while. If you aren't interested in why and how things work, then don't bother to do science.

"Why is there something rather than nothing?" was just one person's choice of phrase. I wouldn't read too much meaning into his choice of words.

Kim Jones

unread,
Jan 5, 2015, 8:37:40 PM1/5/15
to everyth...@googlegroups.com



 

On 6 Jan 2015, at 12:09 pm, LizR <liz...@gmail.com> wrote:

Sounds a bit like sophistry. The question could be rephrased 'why/how does anything exist?' - which is a natural continuation of the questions scientists have been asking for a while. If you aren't interested in why and how things work, then don't bother to do science.

"Why is there something rather than nothing?" was just one person's choice of phrase. I wouldn't read too much meaning into his choice of words.


I submit the following as possible equivalent expressions:

Why is there something rather than nothing (given that nothing would be a whole lot easier)

Why do I believe there is something (would I recognise nothing if I ran into it)

If I believe there is 'nothing' yet still encounter something, was I wrong

Would this be evidence of the co-existence of nothing and something? Dark matter seems to do a good job of being nothing and something at the same time 

Leads to all the classic kids' questions:

Why am I here

Why do I believe I am here

Why am I me and not someone else

Who or what am I

What is the meaning of life (you may perhaps substitute 'consciousness' here)

What is the purpose of life (ditto)

Why does anything make sense 

Does anything make sense

Why do I think things make sense etc...

Why do conscious creatures need to know these things?

Kim

Jason Resch

unread,
Jan 5, 2015, 10:03:42 PM1/5/15
to Everything List
On Mon, Jan 5, 2015 at 7:37 PM, Kim Jones <kimj...@ozemail.com.au> wrote:



 

On 6 Jan 2015, at 12:09 pm, LizR <liz...@gmail.com> wrote:

Sounds a bit like sophistry. The question could be rephrased 'why/how does anything exist?' - which is a natural continuation of the questions scientists have been asking for a while. If you aren't interested in why and how things work, then don't bother to do science.

"Why is there something rather than nothing?" was just one person's choice of phrase. I wouldn't read too much meaning into his choice of words.


Just my opinions, so take with a grain of salt, but if you have any questions about how I arrived at these conclusions, I'd be happy to share.
 

I submit the following as possible equivalent expressions:

Why is there something rather than nothing (given that nothing would be a whole lot easier)

From information theory, it's true that nothing takes less information to describe/specify than something. Surprisingly, however, it takes less information to specify everything than it does to specify something or nothing. Consider your inbox before you receive a message. The space of all possibilities remains open until you add that information, having 0 bits of information leaves open all possibilities. While receiving a "blank" e-mail containing no information requires some information. The uncarved block of marble contains all possible statues, until information is imparted via the sculptor's chisels.

 

Why do I believe there is something (would I recognise nothing if I ran into it)


Certain propositions in mathematics are true. There are simple ones like 2 + 2 = 4, then there are more complex ones, like the 10th Fibonacci number is 34. Then there are even more complex ones, like the 151st step of program Y that computes the Fibonacci number is state X. Then you go further, and say "the Nth step of the program J (which happens to be equivalent to the program that your brain is executing in this moment) believes it is a conscious person living on a planet called Earth" and this is a true statement in mathematics. So ultimately your belief in something has the same justification that 2+2=4 has: it can't be any other way.

 
If I believe there is 'nothing' yet still encounter something, was I wrong

Would this be evidence of the co-existence of nothing and something?

Among all possible universes, perhaps one of them is empty. We can conceive of this universe, just as we can conceive of other universes, but the existence of "nothing over there" doesn't affect the "something over here". No more than 0 affects 1.
 
Dark matter seems to do a good job of being nothing and something at the same time 

Leads to all the classic kids' questions:

Why am I here

In all conscious experiences you index yourself to "here" and "now", but other conscious experiences are no less correct in their belief that "here" and "now" (different from yours) are valid to them. Just as New York is "here" for New Yorkers, and Chicago is here for Chicagoans.
 

Why do I believe I am here

All thoughts exist in mathematical truth, including the one where you believe you are here.
 

Why am I me and not someone else

What evidence do you have that you aren't those other selves to? I believe all thoughts are equally yours.
 

Who or what am I

The universal soul to which all experiences belong.
 

What is the meaning of life (you may perhaps substitute 'consciousness' here)

If by meaning you mean purpose, I'd say different individuals can have different purposes that are important to them. Perhaps the goal for non-Godlike consciousness is to approach/reach Godlike consciousness, and perhaps the goal for God-like consciousnesses is to temporarily forget their true identity.
 

What is the purpose of life (ditto)


What did toy mean by meaning if not purpose?
 
Why does anything make sense 

If things were not ordered/well-behaved/comprehendable then life wouldn't be able to evolve (there would be no order or patterns for it to learn to harness and take advantage of), and there would also be no need for brains (since there would be nothing to make sense of). Therefore most conscious beings find themselves in ordered worlds.
 

Does anything make sense


I think so.
 
Why do I think things make sense etc...


That's part of who you are.
 
Why do conscious creatures need to know these things?


I don't know that we need to, but we're naturally curious, and that curiosity includes to knowing our surroundings, where we came from and where we're going. These questions aren't really any different.

Jason

John Clark

unread,
Jan 5, 2015, 10:21:50 PM1/5/15
to everyth...@googlegroups.com
On Mon, Jan 5, 2015 at 6:50 PM, LizR <liz...@gmail.com> wrote:

> Eternal inflation seems to assume there is something because "there has always been something". However if so, it sidesteps the underlying issue - why is there this (eternal) something? The question itself - and any attempted answer - can't be answered causally.

Eternal inflation can't explain how nothing became something but it can explain how *almost* nothing became something, and that certainly seem like a step in the right direction. A scientific explanation shows how simplicity can produce complexity, or to put it another way exposes the simplicity underlying complexity; and that is why the God theory is such a spectacular failure, the explanation is more complex than the thing it explains. 

It's true that the inflation field as proposed by Alan Guth and Andre Linde isn't nothing, but it's vastly simpler that the universe it created and INFINITELY simpler than a omniscient omnipotent infinitely intelligent conscious being. Perhaps some will want to call the inflation field God, but I don't have a fetish for that 3 letter English word so I won't.   
 
 >The question itself - and any attempted answer - can't be answered causally.

It either had a cause or it didn't, and if it didn't then it was random.

 John K Clark

meekerdb

unread,
Jan 5, 2015, 11:32:32 PM1/5/15
to everyth...@googlegroups.com
On 1/5/2015 4:43 PM, 'Chris de Morsella' via Everything List wrote:


From: meekerdb <meek...@verizon.net>
To: everyth...@googlegroups.com
Sent: Monday, January 5, 2015 4:34 PM
Subject: Re: Why is there something rather than nothing? From quantum theory to dialectics?

On 1/5/2015 3:50 PM, LizR wrote:
Eternal inflation seems to assume there is something because "there has always been something". However if so, it sidesteps the underlying issue - why is there this (eternal) something? The question itself - and any attempted answer - can't be answered causally.

"Sidesteps"? or shows it's an invented issue.  If there were nothing would the issue be why isn't there something?  Why should nothing be unquestionably accepted as the default that needs no explanation?

Nice question. 
No easy answer jumps out as being the obvious answer either. Why the human mind seems to come up with this assumption over and over again across various cultures and periods of history. Why do you think humans seem to accept this by and large as the default base state of everything. Could it be an artifact of the way our minds work? 
The idea of nothing as the default base state of the universe -- before God (or Nature) created everything - seems quite widespread.

I think it's an aritfact of the way we make artifacts.  We take simple stuff like mud or rocks or tree limbs and make complex things like pottery or arrow heads or huts.  So when tribes/cities began to deify their leaders they attributed great creative ability to them.  And so the gods were conceived of as having made the world.  At first the gods made the world out of stuff: water, chaos, blood, feces.  But as every cult magnified the power of their god to show He was greater than those other false gods, god had to make more (heavens, stars) out of less, so ultimately god had to make everything out of nothing - otherwise some other god would out do him. 

And it didn't end there.  The greater a god is the less effort he would have to put out to create things.  Yaweh creates the world by six days of labor so tiring he had to rest.  But now theologians have one upped that and God created the world just by his word.  You might think that is the ultimate and no conceivable god could be greater than that, a god who creates by merely speaking.  But Peter DeVries has noted that "It is the final proof of God's omnipotence that he need not exist in order to save us."


Brent
In the beginning the Universe was created. This has made a lot of people very angry and has been widely regarded as a bad move.
    --- Douglas Adams

meekerdb

unread,
Jan 5, 2015, 11:43:13 PM1/5/15
to everyth...@googlegroups.com
Incidentally, to comment on my own post, this shows that theology is a branch of politics, not science.

Brent

meekerdb

unread,
Jan 5, 2015, 11:56:11 PM1/5/15
to everyth...@googlegroups.com
On 1/5/2015 7:03 PM, Jason Resch wrote:
Why am I me and not someone else

What evidence do you have that you aren't those other selves to? I believe all thoughts are equally yours.
 

Who or what am I

The universal soul to which all experiences belong.
 

What is the meaning of life (you may perhaps substitute 'consciousness' here)

If by meaning you mean purpose, I'd say different individuals can have different purposes that are important to them.

But they can't think about them because then they'd belong to everyone equally.

Brent

Roger

unread,
Jan 6, 2015, 12:06:28 AM1/6/15
to everyth...@googlegroups.com

Chris,

    Hi.  I admit that something and nothing may be more of a comedy gold mine than I first wrote.  It's nothing to sneeze at! :-) Although, I wonder if people who aren't interested in this stuff (e.g. almost everyone) would find it funny?

    It sounds like we're pretty much in agreement on a lot of things.  A couple of comments on your comments are:

1. It sure is hard to visualize the "absolute lack-of-all", I agree.  What I try to do is to shut my eyes and try to imagine the universe and all its volume collapsing down to just my body and then just my mindscape.  Then, I push that darkness of the mindscape off to the side into a little point and try to imagine getting rid of that point.  I've never pushed it all the way away out of fear that it may not be so good for your health, but it helps me think that only once it's all gone, including our mind, do we jump to the outside and see the "absolute lack-of-all" as the entirety of all there is and thus an existent entity.  But, it's possible it's just my imagination

2. You mentioned

"...the set is a pure conceptual entity, it never the less is also imbued with a rich set of operations and properties. Even the empty set is a non-trivial conceptual entity."

I don't think of the existent entity that I used to call the "absolute lack-of-all", which is similar to the empty set, as a conceptual entity because in the "absolute lack-of-all" or the nullness inside the empty set, there would be no mind for it to be conceived in.  It's a real existent entity, IMHO, just like an electron is a real existent entity.  Who knows what's inside an electron.  All we really know is that it's an existent entity.  "Electron" and "empty set" are just names for existent entities.

3. When I was talking about removing all things thought to exist in order to get to the "absolute lack-of-all", I don't think there's still a container left.  Instead, I think that that that "absolute lack-of-all" itself is the container.  That nothingness would be the entirety of all there is and thus the grouping, or container, defining what is contained within.  That nothingness is both what is contained within and the container.

4. In regard to the auto-catalytic nature of the existent entity/empty set, I totally agree.  But, my vote for what the multiplication operation would be is that:

o If the "absolute lack-of-all" is a grouping defining what is contained within and thus an existent entity, a grouping is the similar to a surface or edge defining what is contained within and giving substance and existence to the thing.

o If you have this initial surface, what's next to the surface?  The "absolute lack-of-all".  This new instance of the "absolute lack-of-all" is itself an existent entity next to the surface of the original entity.  In fact, I think new identical  "absolute lack-of-all" existent entities would cover the entire surface of the original entity.  

o Each of the new "absolute lack-of-all" existent entities would repeat the process and you'd have an expanding space composed of these "absolute lack-of-all" existent entities.  

    This would be my vote on the autocatalytic mechanism for how this initial entity/empty set could replicate itself.

    See  you.

                                         Roger   





LizR

unread,
Jan 6, 2015, 2:56:49 AM1/6/15
to everyth...@googlegroups.com
On 6 January 2015 at 16:03, Jason Resch <jason...@gmail.com> wrote:

From information theory, it's true that nothing takes less information to describe/specify than something. Surprisingly, however, it takes less information to specify everything than it does to specify something or nothing.

This is the basis of Russell's "Theory of Nothing"

If anyone still doesn't have a copy...



LizR

unread,
Jan 6, 2015, 3:00:22 AM1/6/15
to everyth...@googlegroups.com
On 6 January 2015 at 16:21, John Clark <johnk...@gmail.com> wrote:
On Mon, Jan 5, 2015 at 6:50 PM, LizR <liz...@gmail.com> wrote:

> Eternal inflation seems to assume there is something because "there has always been something". However if so, it sidesteps the underlying issue - why is there this (eternal) something? The question itself - and any attempted answer - can't be answered causally.

Eternal inflation can't explain how nothing became something but it can explain how *almost* nothing became something, and that certainly seem like a step in the right direction. A scientific explanation shows how simplicity can produce complexity, or to put it another way exposes the simplicity underlying complexity; and that is why the God theory is such a spectacular failure, the explanation is more complex than the thing it explains. 

Yes I've said that myself many times on this forum too. 

It's true that the inflation field as proposed by Alan Guth and Andre Linde isn't nothing, but it's vastly simpler that the universe it created and INFINITELY simpler than a omniscient omnipotent infinitely intelligent conscious being. Perhaps some will want to call the inflation field God, but I don't have a fetish for that 3 letter English word so I won't.   

You seem to be obsessed with God, personally I have no wish to discuss that hypothesis. But almost nothing isn't good enough, so a scientific discussion would be welcome.
 
 >The question itself - and any attempted answer - can't be answered causally.

It either had a cause or it didn't, and if it didn't then it was random.

Causal means an antecendent cause, a cause preceding something in time. The problem with EI is that it needs an explanation for how the entire temporal structure arises, even if it has no beginning, the theory needs to explain why this reality and no other?

This is a fascinating question, and one in the scientific tradition of digging deeper into what's really going on.

Chris de Morsella

unread,
Jan 6, 2015, 3:54:01 AM1/6/15
to everyth...@googlegroups.com

 

 

From: everyth...@googlegroups.com [mailto:everyth...@googlegroups.com] On Behalf Of LizR
Sent: Tuesday, January 06, 2015 12:00 AM
To: everyth...@googlegroups.com
Subject: Re: Why is there something rather than nothing? From quantum theory to dialectics?

 

On 6 January 2015 at 16:21, John Clark <johnk...@gmail.com> wrote:

On Mon, Jan 5, 2015 at 6:50 PM, LizR <liz...@gmail.com> wrote:

 

> Eternal inflation seems to assume there is something because "there has always been something". However if so, it sidesteps the underlying issue - why is there this (eternal) something? The question itself - and any attempted answer - can't be answered causally.

 

Eternal inflation can't explain how nothing became something but it can explain how *almost* nothing became something, and that certainly seem like a step in the right direction. A scientific explanation shows how simplicity can produce complexity, or to put it another way exposes the simplicity underlying complexity; and that is why the God theory is such a spectacular failure, the explanation is more complex than the thing it explains. 

 

Well put Liz. Like most (or at least many) I suppose on this list, I am drawn by the idea of the possibility of an Information Theory of Everything. Inflation has made powerful predictions (on the early expected ratios of elements in the era of nucleosynthesis); it has solved intractable problems for the Big Bang – the smoothness problem. But why stop there?

You put the why not well.

-Chris

 

 

Yes I've said that myself many times on this forum too. 

 

It's true that the inflation field as proposed by Alan Guth and Andre Linde isn't nothing, but it's vastly simpler that the universe it created and INFINITELY simpler than a omniscient omnipotent infinitely intelligent conscious being. Perhaps some will want to call the inflation field God, but I don't have a fetish for that 3 letter English word so I won't.   

 

You seem to be obsessed with God, personally I have no wish to discuss that hypothesis. But almost nothing isn't good enough, so a scientific discussion would be welcome.

 

 >The question itself - and any attempted answer - can't be answered causally.

 

It either had a cause or it didn't, and if it didn't then it was random.

 

Causal means an antecendent cause, a cause preceding something in time. The problem with EI is that it needs an explanation for how the entire temporal structure arises, even if it has no beginning, the theory needs to explain why this reality and no other?

 

This is a fascinating question, and one in the scientific tradition of digging deeper into what's really going on.

 

 

--

Bruno Marchal

unread,
Jan 6, 2015, 4:48:17 AM1/6/15
to everyth...@googlegroups.com
On 03 Jan 2015, at 06:05, 'Roger' via Everything List wrote:

Even if the word "exists" has no use because everything exists, it seems important to know why everything exists. 




But everything does not exist. At the best, you can say everything consistent or possible exist.

Anyway, as I said, the notion of nothing and everything, which are conceptually equivalent, needs a notion of thing. That notion of thing will need some thing to be accepte

It is often ambiguous in this thread if people talk about every physical things, every mathematical things, every epistemological things, every theological things, ...

So, we cannot start from nothing. 

We light try the empty theory: no axioms at all. But then its semantics will be all models, and will needs some set theory (not nothing!) to define the models. The semantics of the empty theory is a theory of everything, but in a sort of trivial way.

Computationalism makes this clear, I think. We need to assume 0 (we can't prove its existence from logic alone, we need also to assume logic, if only to reason about the things we talk about, even when they do not exist). 
Then once we have the numbers, the addition and multiplication axioms, we have a Turing universal system and all its relative manifestations, i.e. all computations or all true sigma_1 sentences, and the physical reality is an illusion coming from the internal statistics on the computations.



How is it that a thing can exist? 

With computationalism, we cannot answer that question, but we can entirely explain why. We need to assume one universal system (be it numbers, fortran programs, or combinatirs, ...). Then the physical is a sum of all the computations. 



What I suggest is that a grouping defining what is contained within is an existent entity. 

That is similar to some comprehension set theoretical axioms. The origianl comprehension axiom (by Frege) was shown to be inconsistent by Bertrand Russell, and this leads to the sophisticate set theory, like ZF (Zermelo-Fraenkel) or NBG (von Neuman Bernays Gödel).
Note that set theories assumes much more than arithmetic. Set theories are handy in math, but is a bit trivial in metaphysics. It assumes too much. It contains Quantum mechanics, and all possible variants, including non linear QM, Newtonian mechanics, etc. 




Then, you can use this to try and answer the other question of "Why is there something rather than nothing?".

You reduce existence to set existence. You will need to assume the axioms of some set theory. It is more precise than Tegmark, but it will lead to the problem of where does those set comes from, and why that set theory and not another. The problem with set is that there is no clear notion of a standard model of set theory. With computationalism sets are just a good mind tool for the numbers or the machines.

Bruno





On Thursday, January 1, 2015 12:17:37 PM UTC-5, John Clark wrote:
On Wed, Dec 31, 2014 at 11:36 PM, 'Roger' via Everything List <everyth...@googlegroups.com> wrote:

>>> propose that a thing exists if it is a grouping or relationship present defining what is contained within.

>> If nothing is contained within then that is very well defined, therefore nothing exists.  Something obviously also exists, but if both something and nothing exist then there is no contrast and the word "exists" is drained of all usefulness.

> What I was trying to get at is that the most fundamental unit of existence and the most fundamental instantiation of the word exists is the existent entity that is, I think, incorrectly called the "absolute lack-of-all".

Existent entity? But something that has the existent property is something that exists, and round and round we go. Once again the word "exists" is drained of all usefulness.

 John K Clark



--
You received this message because you are subscribed to the Google Groups "Everything List" group.
To unsubscribe from this group and stop receiving emails from it, send an email to everything-li...@googlegroups.com.
To post to this group, send email to everyth...@googlegroups.com.
Visit this group at http://groups.google.com/group/everything-list.
For more options, visit https://groups.google.com/d/optout.
It is loading more messages.
0 new messages